Thursday, March 29, 2012

Space Warfare XVI: Origins and Scratch Forces

TS Golden BearNo, the configuration shown above is not proposed for any type of spacecraft. With sufficient stretching, I suppose you might justify it in a setting where ships have a magic drive, land on water, and their sea performance is the main design consideration. (If you'll buy that, which bridge do you want to purchase? The pretty one to Marin, or the massive one to Oakland?)

The vessel is the training ship Golden Bear, originally built for the Navy as an oceanographic survey ship. Her relevance to our topic comes from her oddly hybrid appearance: She looks like a not-quite-warship.


Let us suppose, for purposes of story, that some future era has space warfare, and space warcraft to do the fighting.

For the record, this is by no means an inevitable or even likely prospect, at least through the plausible midfuture, with the exception of mostly ground-based weapons intended to engage targets in Earth orbit. Not because of human moral improvement, but for the same reason Antarctica was untouched by the world wars and cold war: lack of strategic objectives.

But let us suppose it anyway. The question then becomes how these forces are developed in the first place. If they are built by existing or future terrestrial Great Powers the origin question hardly arises. Space forces then take form already fully developed, like Athena from Zeus's headache.

We can imagine Pentagon planners, or their counterparts, drawing up specifications for the spacecraft, while the service branches slug it out politically and bureaucratically to determine whether combat constellations have admirals, generals, or Space Marshals in overall command.

But what if space forces do not develop as simple outward projections of existing military establishments? A variety of scenarios might produce this result. The original international regime in space might be demilitarized, as (apart from ASATs) it is today. And when the apple of discord gets thrown it might not be contested for by terrestrial powers.

These might have more immediate concerns, namely each other. If Brazil and Nigeria are on such bad terms as to duke it out, the theater of operations is much more likely to be the South Atlantic than the asteroid belt.

Or the Earth of 2300 or 2500 may be so balkanized as to have no global Great Powers that would even daydream of space fleets. Or, on the other hand, there may be a world state, or demi-state international regime, with a blue-helmet constabulary but no force configured for major combat. Or if all else fails, the Fall of the Terran Empire could leave a welter of rising colonies, a few centuries later, to improvise their ships and military institutions.

In these scenarios the trouble in space may well begin in space. See hints along these lines (and links to further hints) in an earlier post in this series. But even without delving too deeply into the scenarios we can speculate about zero-generation space forces.

Kinetics, rather than beams, are probably (but not inevitably) the dominant weapon. Extensive space travel is pretty much a precondition here, and space travel is all about throwing weight around, fast and accurately. If you can guide a spacecraft to a destination you have at least a fair shot at steering it on collision course into a target. The basic prerequisites of a target seeker: thrusters, sensors, and guidance package, are available from every space operations boneyard.

They won't compare to the milspec'd version, but they don't have to: Their targets are also jury-rigged.

Multimegawatt laser installations combined with observatory-grade optics are less likely to be sitting around handy. Of course, if you really want lasers you can get around this. Laser-triggered fusion, or laser-boosted cargo propulsion, could make high powered lasers readily available, to be weaponized with an array of oscillating hands. But kinetics are the car bombs of space warfare, available by default.

For much the same reason, a wide variety of civil spacecraft can be pressed into war service. If cargo operations are modular, target seekers can ride on standard cargo clamps. Transport/liner types can carry espatiers, or for that matter the service techs needed to keep all sorts of space hardware in operating conditions.

But - here, finally, is the connection to the Golden Bear - survey ships are particularly well suited to wartime conversion. They presumably have a much more extensive sensor suite than most civilian types - and in space, everyone doesn't see everything unless they have appropriate sensors. They have an onboard mission control suited to managing probes, and, in interstellar setting at least, landing shuttles suitable for planets lacking - or not making available - runways and other surface facilities.

None of this makes survey ships the equivalent of purpose-built cruiser types. They have no specifically defensive features, and while they probably have extended range/endurance they have no provision for combat maneuvers. But with semi-demi-realistic space technology both of these are extremely iffy even for purpose-built warcraft.

Human factor also work neatly. Survey ships have relatively large crews, trained and accustomed to dealing with unknowns and uncertainty. Indeed, at least in principle Trek's Enterprise was a survey ship ... exploring new worlds, new civilizations, boldly going where no man has gone before. In practice, to be sure, she was hardly 'zero generation,' with an armament able to engage Klingon battlecruisers on equal terms, and plenty of redshirts to boot.

The availability of survey ships for war conversion depends on whether they exist. This is by no means inevitable (even given extensive human space travel). The chancy work of exploration may still be assigned to robotic probes, with mission control and the research team staying safely in Pasadena. (Or at least San Francisco's Presidio.)

Survey ships in something like the classic sense work best in settings that - conveniently for authors, if not for the inhabitants - have FTL travel but no FTL radio. Which means that to do any serious exploring, mission control must go with the mission, not stay back on Earth.

Stepping back, other considerations come into play. If the fighting breaks out in a single planet's orbital space - rather than the classic scenario of deep space war, interplanetary or interstellar - providing armament remains much the same, but the focus is likely to be on much smaller craft and much shorter missions.

Survey ships are pretty much irrelevant to such a fight. Indeed, quite apart from the specific topic of this post, combat between local forces in orbital space has a lot of 'air force' characteristics, in contrast to the traditional maritime image of deep space.


Discuss.





Related Links:

Atomic Rockets, of course - especially, but not exclusively, the pages on space warfare.

And previously in the Space Warfare series:

I: The Gravity Well
II: Stealth Reconsidered
III: 'Warships' in Space
IV: Mobility
V: Laser Weapons
VI: Kinetics, Part 1
VII: Kinetics, Part 2 - The Killer Bus
VIII: Orbital Combat
IX: Could Everything We Know Be Wrong?
X: Moving Targets
XI: La Zona Fronteriza
XII: Surface Warfare
XIII: The Human Factor
XIV: Things As They Ought To Be
XV: Further Reflections on Laserstars


Also ...

Battle of the Spherical War Cows: Purple v Green
Further Battles of the Spherical War Cows

Plus

Space Fighters, Not
Space Fighters, Reconsidered?

And, indulging in heresy -

Give Peace a Chance




The image of the Golden Bear comes from its Wikipedia page.

201 comments:

1 – 200 of 201   Newer›   Newest»
Anonymous said...

Hmm, colony on colony warfare...A war between Callisto and Ganymeade over the mining rights to the dozens of minor moons of Jupiter; A response from Earth would take months or years so the two colonies would have to work it out themselves; one puts some Bazooka-like rockets on one of their cargo transport rockets; the other colony responds with an armed fast transport; each colony tries to build more and better armed spacecraft; at some point, they actually start shooting at each other; either one or the other wins, neither wins and the war ends in a draw, or they destroy one another (off-world colonies strike me as very fragile).

Ferrell

Brett said...

I think the Survey Ship's advantages really depend on the type of combat that's taking place. Is this ship-to-ship combat in interplanetary (or even interstellar) space, or assaults launched between "fixed" facilities?

The survey ship's sensor suite is useful in the former . . . to a degree. The problem is that its size also makes it much more of a target, and requires more fuel to maneuver once the enemy starts shooting solid-state lasers at you.

If it's a war between places with "fixed" facilities (like between "The Belt" and Earthspace in some type of interplanetary space opera), then you wouldn't have to stick big sensor suites on your ships. You'd just use the large stationary arrays of telescopes at home to make out the details, then put radar and some smaller passive telescopes on your drone ships to use to find targets in the initial wave.

Kinetics, rather than beams, are probably (but not inevitably) the dominant weapon. Extensive space travel is pretty much a precondition here, and space travel is all about throwing weight around, fast and accurately. If you can guide a spacecraft to a destination you have at least a fair shot at steering it on collision course into a target. The basic prerequisites of a target seeker: thrusters, sensors, and guidance package, are available from every space operations boneyard.

I'm still not entirely convinced of this. You either need a spaceship that's moving quickly relative to its target, or you have to accelerate the projectile (which either accelerates the ship, or requires engines on the projectile). Either imposes some serious power requirements, and to make it worse, kinetics can be deflected. You don't even have to destroy them - just hit them with something that causes them to go flying off-course from the target. That's stuff that's happening today, such as with Israel's Iron Dome system.

I prefer lasers. They're devastating to any sensitive exposed equipment (such as the survey ship's telescopes), and even missing the target can lead to victory by forcing the targeted ship to maneuver until it runs out of fuel. They also make great "fixed installation" weapons if you can keep the beam tight enough, and stick it on an airless planetoid/asteroid to serve as a heat sink. Their only huge problem is the waste heat.

Thucydides said...

Military laser and kinetic weapons might not actually start in orbit; I can visualize large transport aircraft that carry a megawatt laser or high power rail gun that can engage targets from ground level to LEO. Since the packaging is relatively small (sensors, fuel and so on account for a large part of the aircraft's size), powers looking to take the fight into orbit (either to protect existing installations or shoot back at threats) might start packaging these weapons into X-37 sized vehicles. Still no space jock fighter pilots here though. By evolution this makes it the Aerospace Force.

Other places where we might get space forces are laser launch systems. The beam is high power, high quality and already hardened to a certain extent (the ground installation is pretty massive to protect the optical train and keep it aligned during use), the militarization will be an add on of "fighting mirrors" in orbit to provide a much greater arc of fire for the beam. Now we have a large ground crew, but still no people in space. The Air Force or Space Command might be the owners and operators of the laser, while the Air Force, Space Command or even the Navy might own the mirrors (imagine mirrors popped up into position from ships or submarines at sea to get the Navy connection).

KE interceptors for use against asteroids will be fearsome weapons indeed. ORION powered interceptors can accelerate at mind blowing G levels to torch missile speeds and delivers a Gigaton of energy on impact. Some megalomaniac might consider using these as weapons against other space installations, but once again, few to no crews in space. Here ownership could come from the Army, as an extension of interceptor missiles and ABMs.

The only semi PMF scenario that I can come up with to develop a manned space force (and an improvised one at that) might be insurance companies paying private contractors to remove space junk and be prepared to deflect incoming asteroids. These "Kepler Cowboys" will have incentive to make spacecraft with higher levels of performance, and these craft will be equipped with tools and devices (as well as sensor equipment) that could be pressed into service as weaponry. Laser brooms to ablate small peices of space junk and herd them into new orbits could cripple the optics of "opposing" spacecraft, for example. The only issue here is the "Kepler Cowboys" are already doing dangerous work, and might not be too willing to take on offensive operations for King and Country (or become mercenaries).

Brett said...

@Thucydides
KE interceptors for use against asteroids will be fearsome weapons indeed. ORION powered interceptors can accelerate at mind blowing G levels to torch missile speeds and delivers a Gigaton of energy on impact.

What are you shooting at that needs a gigaton-level hit to destroy? You'd be better off just using the nuclear weapons themselves (or better yet, modify them into neutron bombs). You don't even need for them to hit - just get them close enough for the burst of radiation and neutrons to fry the electronics and crew.

If it's just asteroids you need to deal with, better to move them with giant stationary lasers or the gravitational "tug" trick. Or push a rocket up to them and push.

I agree that there's not much use for people in space warfare, at least not in the PMF. Rick's "Survey Ship as Command Platform" idea is the most plausible, where the command station gives orders to a bunch of semi-autonomous drone ships that do the actual fighting. You might want humans, if only to give the authorization for your drone ships to engage the targets in combat.

Hugh said...

Perhaps the starting point for kinetic munitions and drones will be built by the contestants on "Robot Star Wars"?

We're at the point already where it's just affordable for hobbyists to get microsats launched, and presumably in this future the earth to orbit cost has dropped by an order of magnitude.

It's also possible for hobbyists to create rather sophisticated model rockets and UAVs, with the primary limitation being the dislike of other people to risk having bits of hardware fall out of the sky onto their heads or homes.

Some people will use the possibilities of cheaper space launch for noble goals like crowd-sourcing the operation of Mars rovers and the like. Others will just want to have their robots bash one another to pieces. And they'll probably find a whatever the future equivalent of a TV production company is to sponsor them.

Byron said...

This is a topic I've done a lot of thinking on, and it leads to some interesting places. First, it's highly unlikely that any space warfare will begin except between space-based powers. Powers on Earth either do not have ASAT capability or have too much to lose. In either case, it makes no sense to devote the resources that would be required.
To deal with situations between a colony and a planet, chances are that economic means, due to the power of the nation in question, will suffice. Once the first shot is fired, all bets are off.
The exact means by which said first shot is fired is the big question. If the above theory is correct, and humans are humans, then any space force will not be fully formed. I can see reasons for all sorts of things that we normally dismiss. Space fighters make a good example. For one thing, lancers are the easiest form of space weapon to make, and work best when the launcher is small and maneuverable. Also, the power that builds the fighters can keep their larger vessels available for trade while still being able to use them for power projection.
And Thucydides, I really doubt that we'll see these Orion interceptors, or a handful at most. They're simply too dangerous.

H said...

Well. I think it is highly probable that space warfare starts on earth orbit.
Precisely because the high value of all space based assets makes them highly attractive targets, which are relatively easy to take out if they are not protected.

Any conflict which includes at least one power with important space capabilities will see the other trying to somehow eliminate that advantage.
(Think for example a possible US-China conflict over Taiwan, it seems unreasonable to supose that China wouldn`t try to eliminate the advantage that the US has in space based comunications and reconnaissance).

If space warfare capabilities have to be improvised, wether in a conflict between colonies or Earth powers, kinetik projectiles will be the first weapon of choice. Simply because if you can move through space you already have everything you need.
Lasers might be tried later on, particulary if the conflict happens between traditional militaries.

Since most spacecraft are already automated most space warfare in range of a groundstation will remain that way, with commandships wether improvised or purpose-built, used as mobile control stations in order to avoid lightspeed lag in operations far away.

Just my humble thought.

Byron said...

H:

Any conflict which includes at least one power with important space capabilities will see the other trying to somehow eliminate that advantage.
(Think for example a possible US-China conflict over Taiwan, it seems unreasonable to supose that China wouldn`t try to eliminate the advantage that the US has in space based comunications and reconnaissance).

I'm not so certain of this. Both sides have important space-based capabilities, and if they start taking out ours, we'll start doing the same to them. At the moment, it might make sense for China to do so, but what about 20 years from now? And then things like debris enter the picture, because people tend to get cranky when their satellite TV is interrupted.

Hugh:
Perhaps the starting point for kinetic munitions and drones will be built by the contestants on "Robot Star Wars"
Interesting idea, but not likely, IMO. The regulatory environment is not going to approve of such things. Plus, what's the point? Fighting robots can be done a lot more cheaply on earth, and space doesn't seem to offer any compelling environmental advantages.

Andy Strauss said...

The ships we know and can construct today are poorly "spaceworthy". As vessesl from 1400s were poorly "seaworthy".
If or when we have strong colonies on other moons and planets in this system (Jovian Chronicles - like) then those colonies might want to defend themselves.

If resources are scarce, then oposite forces would prolly try to capture, rather than destroy, leading to a more "infantry" type of combat.

As to the initial assumptions of would a scratch spaceship be a converted scout, I guess so. At the same time I don't think any ship in the battle of Salamis were scout ships of the time.

Anonymous said...

(SA Phil)

I think a vessel from 1400 was far more seaworthy than our current spacecraft are spaceworthy.

I think a better comparison would be 1400 BC seacraft.

That said I dont think true Operatic performance levels will ever evolve. So we will never reach the equivilent of "modern day" seaworthiness in space.
============


Looks like purple vs green is back -- heh --- Which always seems to me to be related to distance between combatants. Its hard to beat the Lightspeed weapon.

==============

As to Rick's point about Major Spacefaring powers -- the old Traveller 2300 game had an interesting take on this with colonial powers based on Modern Earth Nations.

Byron said...

Part of the issue is that from the beginning of real shipbuilding (beyond the level of dugout canoes) we've had dedicated warships. Always. Greek galleys were built for war, and while converted merchantmen have been used, that has mostly been during times when civilian and military designs were very similar, such as the age of sail. Other times, such as Ancient Greece and now, the difference is enough to render it unworkable. The degree to which military and civilian vessels will separate, and how quickly, is another interesting question.

Thucydides said...

Why would we want an ORION asteroid interceptor rather than a gravity tractor or other method? Mostly to do with reaction time:

http://nextbigfuture.com/2009/02/unmanned-sprint-start-for-nuclear-orion.html

"Get to high velocities with only a few explosives and small shock absorbers or no shocks at all. Launch against a 100 meter chondritic asteroid coming at 25 km/sec. 1000 megatons if it hits. Launch when it is 15 million kilometers away and try to cause 10000km deflection. A minimal Orion weighing 3.3 tons with no warhead would do the job. 115 charges with a total of 288 kiloton yield. Launch to intercept in 5 hours. Ample time to launch a second if the first failed."

If you detect the incoming close in, only an ORION interceptor will be able to engage. For various reasons, I can picture the squadron being maybe five interceptors in their hangers and a distant launch bunker. This might be co located with the current US ABM interceptor squadron in Alaska for political and security reasons.

I am also rethinking the idea of how a C&C ship might operate. I had thought, given the huge size of weapons platforms in the Implausible mid future (laserstars needing vast radiator arrays and liniacs a kilometer long) that small C&C vessels might be carried along as parasites, casting off as the platforms moved into combat range (light seconds away!). Since the C&C is such a vital part of the constellation, it will be worth everything to destroy or disable, and once identified overwhelmed with swarms of KE weapons and concentrated laser fire. Hardly a comfortable place for the Admiral and his staff.

With this in mind, the C&C might be a massive vessel in its own right, something like a Mars Cycler which can carry active and passive protective arrays, as well as the internal size and life support capabilities to keep the crew comfortable and sane during normal cruise.

Byron said...

Thucydides:
That's why I said a few Orion interceptors. Maybe two or three in operation, and a few more disassembled.
Actually, I don't see any call for them at all. By this point, we will have sufficient detection capability to pick up anything of this size well before it could become a threat. No stealth in space applies to asteroids, though not quite as strongly.

I am also rethinking the idea of how a C&C ship might operate. I had thought, given the huge size of weapons platforms in the Implausible mid future (laserstars needing vast radiator arrays and liniacs a kilometer long) that small C&C vessels might be carried along as parasites, casting off as the platforms moved into combat range (light seconds away!). Since the C&C is such a vital part of the constellation, it will be worth everything to destroy or disable, and once identified overwhelmed with swarms of KE weapons and concentrated laser fire. Hardly a comfortable place for the Admiral and his staff.
I agree that the C&C ship would be well outside range, though not with the parasite part. That hinders squadron performance.

With this in mind, the C&C might be a massive vessel in its own right, something like a Mars Cycler which can carry active and passive protective arrays, as well as the internal size and life support capabilities to keep the crew comfortable and sane during normal cruise.
What is it with you and cyclers? Under these sort of circumstances they are useless. Not use-impaired, useless. As for size, take a normal laserstar and replace weapons with crew and defenses. We did this last year, remember?

jollyreaper said...

Couple of comments.

First off, damn you, Google, for continuing to screw up this product. You ruin everything you touch. I want subscribe by email back!

I think if we're trying to make a realistic scenario for war it will have to be some manner of "fall of the empire" scenario, even if it isn't an express empire but an understanding between nations and mutual cooperation.

jollyreaper said...

On a tangentially related note, some speculations I made concerning realistic starfighters. I fear it might be a bit of a text wall but if there's any place on the web it's suitable... maybe here? lol

We all love the big, shooty WWII in space starfighters and starships. It doesn't really hold up under any kind of analysis and thinking about it too hard will make your head go splody Scanners-style. Even if you're able to suspend disbelief while watching Vipers and Raiders battling in deep space, your brain starts hurting when you see them flying through an atmosphere and struggling to get each other in range of projectile guns and you realize either opponent could be utterly smoked by a modern F-18 firing AAMRAM's from 40 miles out. (And the long-range version is supposed to be able to reach out to 100mi.) You've got self-ware freakin' robots and FTL and engines that can take a fighter from surface to orbit but guided missiles are busting your balls? Stop thinking, stop thinking...

So, how do we back into the kind of setting that allows for the kind of coolness we want without having to suspend our disbelief by the neck until dead? Here's my stab at it.

So, we suppose there exist rifts that can be breached by vessels using field generators. They are randomly distributed around planetary bodies but only the ones in the air are easy to access. You pop through them and are in an empty void. It's like space in that it's pretty empty and dark, no gravity, no air but there aren't stars or anything else like that, it's pretty empty. However, there are clumps of matter to be found that are rich in a great variety of mcguffinites. What's more, if you venture out far enough you might find another rift and find yourself on an entirely new world.

Now the difficult part with the rifts is that you have to be in the air to get to the good ones. Trying to go through on the ground is cumbersome, especially on the return since you could smack right into the dirt. You need wiggle room. So the first vessels to pass through a rift into a void are airplanes. But you can't just slap a field generator on a bog-standard plane and send it through. No, it has to also double as a proper spaceship; pressurized cabin, proper life support, thrusters, reaction control system, radiators, etc. It can fap about in the void and upon return must be ready to hit a wall of air and restart the air-breathing engines before whacking the ground.

jollyreaper said...

So, once you're out in the void, you find mcguffinite. What does it let you do? Ah, lots of things. The big assumption here is an atomic power source and some kind of refined handwavium that, combined with ridiculous amounts of power, gives you antigravity.

Now you can build massive, hulking ships that can hang there in the air all impressive-like and also move through the void.

Nuclear reactors are just barely affordable for superpowers and only for special circumstances. They need to be quite a bit cheaper for our ships. Aside from that, the tech can feel pretty 20th century.

The void ships float across planets to reach new rifts. They compress air for use in the void just like a sub. They'll need reaction mass. Do they liquify oxygen and hydrogen to store onboard and use like our current rockets or do they take on water and run it across a bare reactor core to make a nuclear steamjet? Don't know which idea makes more sense. How do they move in atmosphere? Either the antigrav field can be made asymmetrical allowing the ship to move in a direction or else they'll need to use air-breathing jets. Perhaps the main engines can work as air-breathers in atmosphere and pure rockets in the void.

Once in the void the ship aligns on the beacon for the next rift, fires up the engines and gets up to cruising speed. Because the ships have limited delta-v, the top velocity won't be ridiculous. Nobody is in orbit, speeds are slow. There's no grace to them, nothing to inspire poetry and fancy. The ships look like ugly, blunt instruments and move ponderously. They would be completely useless in conventional space but are capable enough in the void.

Ships will need to kill velocity before making the next transition because they are slamming into atmosphere and some rifts are pretty close to sea level.

So, you have trade between civilizations across the void. You have mining activities in the void itself. Mining stations need defenses. Likewise, rift points will be defended by battlestations specially constructed for the task.

If we assume that sensors are futzy in the void, radar tracking could be problematic for long-range guided weapons like cruise missiles. The most reliable weapon might remain the big gun with the shells using proximity fuses. Guns would have an unlimited theoretical range but a shorter effective range. Fighter craft can be launched by larger ships to chase down smaller, faster targets and also screen the ship from enemy fighters.

The other factor in favor of guns is that they remain effective in atmosphere. Warships cannot afford to be optimized for one environment over another because they are often called to fight in both. Fighters, of course, can be specialized for atmosphere or space and those that try to fight well in both will suffer the penalty of the generalist.

The only other assumption required is that a bit of a tech plateau has been reached, a decelerando. This provides a good, long time for humans to spread through the territories of the void. What is the void? Failed universe, strange dimension, something else? Impossible to say. Each void seems different. As to the other planets, are they even in our same universe? The stars are all different but so many of them feature biospheres compatible with human life. How could this be? Mysteries within mysteries.


A few more thoughts about ship design. Depending on how fast they move in atmosphere, streamlining could be effective but also consider how effective a tactic it is to come through a rift at high speed. An armored military ship might be able to slam across at multiple machs. If it isn't torn to pieces it could provide the same kind of fireworks as the Galactica's atmospheric jump. I'd still like to limit the atmospheric top speed to under 100 mph.

jollyreaper said...

If such tactics are used, ships would require sturdy superstructures with anything delicate retractable. Of maybe it's simply not practical for the big warships, sort of like how the Italians built a submersible cruiser that never worked right. (looked like a sub with stacked big gun turrets like a cruiser.)

Fleet fighters would need antigravity so they can operate in atmosphere and land properly on the ships but terrestrial fighters could do away with that excess weight but require proper runways.

One other thought is it'll be interesting how you can go topside on these ships in atmosphere and there would be walkways and railings and the airlocks would be open on both sides but securing for the void would require making the ship airtight just like prepping a sub for a dive. You could still take a suit and go for a walk outside in the void but I would wager that would be rather unsettling.

As for navigation in the void there would be beacons but the backup would be an inertial navigation system, just like our subs use. Of course, they drift and lose accuracy so need navigation fixes every so often. If a ship is trying to find an unmarked rift, fighters might have to me sent out to scout the vicinity since the rift might not be very detectable from a distance.


I really liked Charlie Stross' Missile Gap. Earth appears to be peeled like a grape and laid down flat on a disc that is the diameter of a solar system. Beyond the confines of our own familiar continents stretch billions and billions of miles worth of new territory. It becomes implied that there are a number of parallel human civilizations placed on these continents by experimenters who remain unknown. Many dead civilizations as well as empty continents are found. Nuclear-powered colony ships are sent out to colonize.

It would be pretty wild to have the same thing play out. Expanding on the idea I'd mentioned earlier, there should be no proof of experimenters but lots of compelling evidence that the environment is not natural.


A scientifically-minded character could posit that the worlds seem like experiments and the voids are like barriers to be raised and lowered as with rats in a cage maze. Isolate populations for whatever reason and bring them into contact at a suitable time for purposes beyond calculation.

I figure that there will be commercial routes that are well-known and easy traveling but also unusual routes that take a long time and go interesting places. A month out from one rift gate could be another rift leading to a dead world and two days of travel across the cratered and irradiated landscape is another rift that leads to another void and through there another void and six months later you have found a back way in to an enemy's important colony world.

I figure the major rifts would be defended on both sides, in the void and on the ground, so hitting a fortified world is tough. The fights would be for empty worlds or ones with inferior natives.

jollyreaper said...

And again the scientists marvel at the distribution of humans and human-friendly ecologies. Explorers could try combing the wreckage of the destroyed worlds to find samples of newer technology. But there's the uncomfortable conclusion that they rarely find examples of worlds much more advanced than their own because they're all ruins. Did they destroy themselves or did someone else?

An empire could stretch across dozens and dozens of habitable worlds. The situation of Rome at its height is dwarfed. A message by courier with horses to swap could reach from Rome to Britannia in what, weeks? Radio can transmit across planets but only a vessel could cross the rift and then use relay beacons to cross the rift where radio fizzes out in a short while. Maybe it would take a week to transmit a message across the empire. But to move a person from the capital world to the border? I'm imagining a nuclear-powered aircraft. We played around with the concept in the 50's. Imagine something like a nuke-powered Concorde staying in the air and in the void for three months to make it to the furthest reaches of the empire. It would have to have some size for comfort but would still be fairly cramped. Enough food for the trip, drinking water condensed out of the air, the nuke plant would be good for years. Mach 2 or 3 in atmo, some ridiculously high speed in the void, blowing half the remass to get up to speed and half to slow for transiting the other rift, likely the fasted thing you'll ever see in the void.

I think the religious implications of a system like this would be pretty mind-bending. I think the simplest approach would be to assume that there's intelligent design and thus a creator god, assume that he's got our best interests at heart and worship blindly. The more scientific would not be able to refute the evidence but be skeptical about the benign nature of those responsible. Naturally, that kind of heresy is not spoken aloud in polite company.

Aerospace fighters would have some interesting characteristics. They'd need air-breathing engines for atmo flight. Fine, we know how to build jets. But they'd also need an additional powerplant for providing enough juice to run the antigrav rings. If they can't just redirect the jet exhaust past a generator to power them, they might need an auxiliary turbojet just to run the generator. It would fire up any time antigrav engaged. Once in space, either set of engines is no longer operational so fuel cells would be required.

If we really wanted to make the aerospace fighter complicated, imagine if the aircraft was built like a layout four-engined BSG viper. In air mode the engines and gun pods are flush with the fuselage. When transitioning to space, the engines swing out on movable sponsons and you now have the engines in an X configuration like a Starfury. Guns also swing out and are gimbaled so that they can now shoot targets far off of boresight. This supposes there's enough of a maneuverability advantage to offset the mass penalty of the extra machinery to make the transformation possible.

Either way, a pure void fighter or a pure atmo fighter would surely have the edge over a hybrid. The versatility would potentially be worth the penalty.

Thucydides said...

The initial idea was the crewed C&C ship, being much smaller that the laserstar, would benefit from riding attached until it was time to deploy the constellation. There might be multiple laserstars, each a kilometer long, and one C&C ship about 40m in diameter attached to one of the laserstars.

A cycler is the size factor for a large, robust and crewed ship operating under these circumstances. If you are setting up the Earth Space NAvy to patrol between Earth and Mars then it would actually operate as a cycler, but in the more general sense I am talking about something more like a mobile habitat rather than a traditional spaceship.

kedamono@mac.com said...

My take on this is that most combat in the PMF would be "littoral" combat. That is, in orbit about a world. Given the constraints of fuel and mass, it's highly unlikely you'd send a fleet out to intercept another fleet, only to have most of your ships ending up on 10,000 AU elliptical orbits for the next century or so. If they're not blown up first.

Combat is going to be orbital, if happens at all. And here the home team has the advantage. They can mine LEO with retrograde orbiting satellites. On command, a small charge breaks the satellite up into a million pieces that then passes through the enemy fleet at 7.8 km/s, about 2.6 Ricks of impact energy for every kilogram of satellite debris.

Then you have ground based laser bases, who don't worry about radiating heat from their terawatt arrays. Add in bomb pumped X-ray lasers satellites, and it's murder in LEO for anyone stupid enough to pull into orbit.

So what do you do as the attacker? Sadly, I'd say hit the planet with impactors. Doesn't have to be a big object, just big enough to pass through the atmosphere, if there is one, and impact on the planet. I wouldn't aim for civilization centers, you want to take those over, but for open areas to demonstrate that if pushed, you can dinokiller that world if need be.

Brett said...

@Kedamono
So what do you do as the attacker? Sadly, I'd say hit the planet with impactors. Doesn't have to be a big object, just big enough to pass through the atmosphere, if there is one, and impact on the planet. I wouldn't aim for civilization centers, you want to take those over, but for open areas to demonstrate that if pushed, you can dinokiller that world if need be.

In that type of setting, I doubt the planet is just going to be sitting there with fixed defenses. It will probably have its own defense fleet that it can send out after someone accelerating kinetics in interplanetary space (or even launch and send one-way drone ships to attack).

I tend to think of planets in such a setting as shore batteries. Obviously limited, but still very useful for protection.

kedamono@mac.com said...

@Brett
In that type of setting, I doubt the planet is just going to be sitting there with fixed defenses. It will probably have its own defense fleet that it can send out after someone accelerating kinetics in interplanetary space (or even launch and send one-way drone ships to attack).

I tend to think of planets in such a setting as shore batteries. Obviously limited, but still very useful for protection.


Yes, but I doubt very much it will be a manned fleet. UAVs in orbit yes, guys getting bored waiting for something to happen, no. Last thing bean counters want in orbit is something that costs money and eats resources. A UAV just sits there in orbit till you need it.

A robotic fleet allows you to have a first strike capability, and hopefully you can shoot down any inbound impactors. As Rick pointed out, if the inhabitants use lasers to launch stuff into orbit, then they are good at pinpoint aiming with their lasers. The will probably be booster lasers in orbit, so there's no shortage of fire power. The Kzinti Lesson is a hard one to learn.

Byron said...

Jollyreaper:
First off, damn you, Google, for continuing to screw up this product. You ruin everything you touch. I want subscribe by email back!
Absolutely seconded. I'm getting very annoyed having to check manually with the usual pace of these things.

The scenario you outline is interesting, and falls under the category of "well-explained implausible." It's a little outside the general scope of this blog, but it leads to quite interesting places. Go for it.

Thucydides:
The initial idea was the crewed C&C ship, being much smaller that the laserstar, would benefit from riding attached until it was time to deploy the constellation. There might be multiple laserstars, each a kilometer long, and one C&C ship about 40m in diameter attached to one of the laserstars.
But in that case, it slows the laserstar it's attached to. Fold in all the support facilities and defenses, and make it the size of one of the laserstars. Or just a separate ship. Either way, the performance hit is removed, and it can run on its own at need.

A cycler is the size factor for a large, robust and crewed ship operating under these circumstances. If you are setting up the Earth Space NAvy to patrol between Earth and Mars then it would actually operate as a cycler, but in the more general sense I am talking about something more like a mobile habitat rather than a traditional spaceship.
Then say "mobile habitat" not "cycler" please. You always bring those up, and the rest of us always point out that they're quite useless. As a reminder, a patrol cycler is actually worse then useless, as you've constrained yourself to fixed windows, and given how much it cost, you don't have the rapid reaction force I do.

kedamono:
My take on this is that most combat in the PMF would be "littoral" combat. That is, in orbit about a world. Given the constraints of fuel and mass, it's highly unlikely you'd send a fleet out to intercept another fleet, only to have most of your ships ending up on 10,000 AU elliptical orbits for the next century or so. If they're not blown up first.
I agree to a limited extent, though I expect that major fleet battles will happen far enough out that the planet itself won't be a huge factor. I'm talking "edge of hill sphere" or maybe "near deep space".

Combat is going to be orbital, if happens at all. And here the home team has the advantage. They can mine LEO with retrograde orbiting satellites. On command, a small charge breaks the satellite up into a million pieces that then passes through the enemy fleet at 7.8 km/s, about 2.6 Ricks of impact energy for every kilogram of satellite debris.
Check your math. 7.8 km/s is actually 6.76 ricks, and that's if a vessel in orbit hits a stationary object. For a closing collision, the number is 27.04.


Again, not so sure. I'm skeptical of bomb-pumped lasers, and an attacker can destroy the orbital infrastructure without venturing into LEO. For a more detailed discussion of this, see Space Warfare I and XII.

Anonymous said...

=Milo=



kedamono@mac.com:

"On command, a small charge breaks the satellite up into a million pieces that then passes through the enemy fleet at 7.8 km/s, about 2.6 Ricks of impact energy for every kilogram of satellite debris."

If you want to mine your orbit, you have to mine it in both directions. Your attackers are free to enter a retrograde orbit if they wish - if they're coming in from being orbit, then neither a prograde nor retrograde insertion is harder than the other.

But the real problem with mining orbit is that you're then stuck dealing with your own minefield.


"So what do you do as the attacker? Sadly, I'd say hit the planet with impactors. Doesn't have to be a big object, just big enough to pass through the atmosphere, if there is one, and impact on the planet. I wouldn't aim for civilization centers, you want to take those over, but for open areas to demonstrate that if pushed, you can dinokiller that world if need be."

That sounds like the equivalent of trying to get an enemy military to surrender by shooting a 5.56mm rifle bullet into a tree.

Okay, fine, firing a rocket-propelled grenade launcher into a tree.

Dinokillers are hard. If you can afford to move around asteroids that large, you can afford to give your spaceships more maneuverability.

francisdrake said...

Exploring the solar system is done by robotic explorers. Given the astonishing speed how fast electronics evolve, there seems little need for future manned explorers within our solar system, where mission control is never farther than a few light-hours away. 'Exploring the Unknown', which made all the stories of the historic explorers so thrilling, does not seem to be exactly a human task in space.

If humans go somewhere in space they will go on purpose. The first expeditions to Mars or Jupiters moons do not really qualify for armed conflicts. Looking one step further into the future still does not give too many reasons to bash each others heads, as the Antartica analogy applies. A more distant future with significant political (colonies fighting each other) or economical (big bad mega corporations) reasons is required.

Coming back to Rick's original scenario involving jury-rigged vessel and makeshift weapons seems to be much more demanding on creativity than fleets of ironsides in space. They force the author to think out-of-the-box. "How could a relay satellite be used as a reconnaisance drone? Can I direct it into an enemy vessel to use it as a cheap slug weapon? Can a man in the airlock with a gun in his hands damage a nearby vessel, without the gun seizing up due to lack of cooling?"

The engine can be used as a weapon, even without being the notorious Orion 'we bomb everything'-drive. The most feeble interplanetary engines will be a source of hundreds of megawatts of thermal and radiation power output. Thermal energy will dissipate pretty fast, but pointing your tail to an approaching enemy ship will shower it with lots of gamma rays and neutrons, provided some kind atomic, fission, fusion, antimatter, whatever drive is used. As a bonus your own shadow-shield will help protecting yourself from enemies emissions, in case he wants to play the same trick on you, and it points your heat radiators edge-on to him, which makes them somewhat harder to hit.

Given the concept of modular spaceships, it might be feasible to combine a standard engine (maybe an explorer has double engines for extra reliability), standard
structures, tanks and radiators with an armored crew cabin, a mil-grade sensor suite and cargo containers sporting unobtrusively a missile-launch system inside. While not actually being the star-destroyers of choice, these vessels might be an affordable way to add some punch to interplanetary relations.

Hugh said...

Given the setting in which missiles and drones dominate, I'd expect to see a lot of civilian hulls with milspec launchers and lasers bolted on.

Byron is right that we've always had warships...but there have also been long periods when the hull and 'drive' of a warship was largely indistinguishable from a merchant. Even in the heyday of the galley they weren't used in the Atlantic, or the Chinese coast for that matter.

Increasing construction costs and cheap and deadly missiles at sea have brought about Iranian Boghammars, Royal Navy ARAPHO conversions, and an increasing number of warships with hulls built to mercantile rather than milspec. If the missiles hit you're dead, so go with the quicker and cheaper solution.

And we're just starting to see this in the air with the P-8, a Boeing 737 with a bomb bay. And while the US plans to use them only for anti sub patrols, you can bet that other nations who don't have fleets of B-52s are going to buy P-8s and load them up with JDAMs, Harpoons or even AMRAAMs.

I'm not saying that these improvised warships will be as good as purpose-built warships, but they are likely to be good enough and available now, in numbers.

Cambias said...

Reaper: your "rifts up in the air" scenario can mean only one thing -- large airships capable of reaching the rifts. STAR ZEPPELINS!

jollyreaper said...

It won't feel quite like space Zeps because the ships are far more robust. It really would feel a bit like WWII in space and in the air.

If we run with the idea that the worlds are all experimental human societies and Different histories are allowed to play out, consider how WWII Japan was the result of a primitive society rapidly upping the tech to parity with the west. You had the weird clash of modern tech and primitive thought.

Imagine the same thing for an Aztec society. They develop modern war tech and start conquering. While they may not literally believe blood fuels the sun, the leaders might feel that the terror of providing sacrifices keeps the vassal nations cowed. And the nobility must have war to justify their existence.

So you have a conquering people waging war for religious and cultural reasons, mass human sacrifice as dominance displays. Ambassadors from vassal proles have to journey back to the capital city to see the annual tribute go to their deaths at the sun temple.

I think that would make for a pretty terrifying enemy.

And @byron, yes, it's all about trying to simplify the setting for plausibility's sake. Plausible mid-future stories are very hard to tell credibly and science fantasy like Star Wars falls apart the moment you think too hard. I'm trying to come up with something that could survive a good thinking.

jollyreaper said...

@hugh

I've been wondering about smart weapons like that myself. Our current trend is towards traditional weapons being impossibly expensive. We are going to spend trillions on that idiotic F-35 while highly capable drones are available for millions.

I'd be curious to see what a proper fighter would look like if we were in a national struggle. No more profiteering and bs, everyone from CEO to engineer to riveter knows that death is on the line. Build as good a fighter as possible as quickly as possible in as great a quantity as possible and we are taking kids right out of high school to be combat pilots.

I have a feeling much of the dysfunction is a result of political corruption and peacetime budget larding along with crappy corporate management rather than solely the result of technical complications. It's like the Russians couldn't build a good dreadnought but that doesn't mean nobody else could do it either. The Japanese proved they could do it at Port Arthur.

jollyreaper said...

Oh this is even worse. The comments are threaded now? I'll have to scan up and down the whole bleeding comments section to find the new ones.

Damn you, google!

Rick said...

Ferrell's introductory comment is wonderfully evocative of that old Disney 'Story of Flight' animated film, and how it portrayed the beginnings of air combat in WW I. (Escalating from 'donkey ears' to pistol shots, and on from there!)


I think the Survey Ship's advantages really depend on the type of combat that's taking place. Is this ship-to-ship combat in interplanetary (or even interstellar) space, or assaults launched between "fixed" facilities?

Ah, the wonders of hidden assumptions! As later comments noted, 'survey ships' have little to no place in Solar System exploration, or for that matter interstellar exploration if you have robust FTL comms.


Purple/green - Remember that in this discussion I am considering a special case, where advanced space technology is widespread but advanced laser weapon technology has not (yet) been developed. In general it is easier to jury-rig a target seeker than a combat-capable laser.

But if high-power lasers are part of the standard space tech, it is a different matter!


Rifts - this is a highly operatic setting ... but no more so than any setting with FTL. That said, I agree with Cambias - this is made for star zeppelins!


Other than that, carry on!

Byron said...

NO!!!!!!!!
Why did someone have to bring up zeppelins? They have antigravity, so there's no need for them.

Milo:
I expect never to see someone shooting out of an airlock. A human isn't very good at precise shooting, when all is said and done. If it does happen, it will only happen once.

Hugh:
And we're just starting to see this in the air with the P-8, a Boeing 737 with a bomb bay.
No, we've seen this for a long time. There have been numerous 707 variants in military service (granted, they didn't have bomb bays) and the P-3 Orion was a converted airliner. For some roles, civilian aircraft work well. For others, not so much.

Jollyreaper:
It's like the Russians couldn't build a good dreadnought but that doesn't mean nobody else could do it either. The Japanese proved they could do it at Port Arthur.
[pedantic]Technically, there were no dreadnoughts used during the Russo-Japanese War. HMS Dreadnought wasn't laid down until just after the end of the war. That isn't to say that Russia had more success building pre-dreadnoughts. Of course, the worst Russian ship ever had to be the dreadnought Sovetskaya Belorussiya, which was canceled due to defective rivets.[/pedantic]

Byron said...

francisdrake:
The engine can be used as a weapon, even without being the notorious Orion 'we bomb everything'-drive. The most feeble interplanetary engines will be a source of hundreds of megawatts of thermal and radiation power output. Thermal energy will dissipate pretty fast, but pointing your tail to an approaching enemy ship will shower it with lots of gamma rays and neutrons, provided some kind atomic, fission, fusion, antimatter, whatever drive is used. As a bonus your own shadow-shield will help protecting yourself from enemies emissions, in case he wants to play the same trick on you, and it points your heat radiators edge-on to him, which makes them somewhat harder to hit.
Not so much. Exhaust disperses quite quickly. I remember something like 2 degrees being standard for ion thrusters. Thus, for every 30 meters you go out, the exhaust spot will be about 2 meters bigger. I really doubt you'll be within a kilometer of your opponent, which comes out to about 66 meters. At that range, a 1 GW torch will produce 292 kW/m2. That's quite a bit, but at point-blank range in combat.
I guess my view of the Kinzi lesson is that it's sort of like a bayonet. I don't want to get stabbed by one, but it only works at melee range, and it's not significantly more lethal then normal weapons.

Byron said...

As an addendum to the above, at around 8.5 km, the flux of the engine wash is equal to that of sunlight at Earth. So the range is very, very short.

Thucydides said...

Cyclers are quite useful if there is interplanetary traffic, so the may well become the form factor for any large ship simply because of the existing infrastructure. If we are going with Rick's initial assumption of improvised fleets and weapons, a Cycler is already a mothership of sorts, with a built in fleet of shuttles capable of atmospheric entry. In many ways a Cycler is a better starting point for an improvised fighting platform than almost anything else.

Parasite C&C ships attached to a massive laserstar really affect performance during cruise operations. They are explicitly designed to cast off as the constellation moves into attack formation, in which case all the laserstars are moving with similar levels of performance. A large independent C&C provides mre protection, although at the cost of being easier to spot.

Smart or even brilliant weapons are used here on Earth to pick out targets against background clutter, and engage them without needing a person in the loop. This isn't such an issue in space to space combat, but becomes something of an issue in orbitall operations and a big issue if you need to attack planetary targets from orbit. Givn the vast amount of inexpensive computing power avaiable today, by default most weapons will become "smart" weapons, and for a space fleet, that makes logistics easier (you only need to carry one type of missile, for example. The damage mechanism is always KE, and the fact the weapon is mostly a solid block allows it to pass through an atmosphere.

Byron said...

Thucydides:
Cyclers are quite useful if there is interplanetary traffic, so the may well become the form factor for any large ship simply because of the existing infrastructure. If we are going with Rick's initial assumption of improvised fleets and weapons, a Cycler is already a mothership of sorts, with a built in fleet of shuttles capable of atmospheric entry. In many ways a Cycler is a better starting point for an improvised fighting platform than almost anything else.
No, they're not. Cyclers work only when the cost of delta-V is high. As it drops, which it will by this point, they will no longer be used. As for form factor, I'd go with smaller ships. A cycler has long-term life support, which is only needed for a space station, not a PMF ship. The ship can make the journey before such long-term life support is needed. And remember that the window of opportunity for a cycler is very narrow, and predictable. Neither is a good trait.

Parasite C&C ships attached to a massive laserstar really affect performance during cruise operations. They are explicitly designed to cast off as the constellation moves into attack formation, in which case all the laserstars are moving with similar levels of performance.
This is exactly my point. Standard PMF ships are not maneuverable in combat. Nor is any space vessel that fights under the same drive it cruises under. Cruise performance is quite vital.

A large independent C&C provides mre protection, although at the cost of being easier to spot.
There's no stealth in space. And you could figure out which laserstar it was attached to quite easily.

jollyreaper said...

There are two big advantages of the rifts setting. The first is that you're no longer dealing with schizotech questions of how to justify interstellar warships and warfare. There have been many complaints about settings that are basically next Sunday AD save for the FTL. The economy, society, politics and conflicts can still be like what we are familiar with as a starting point and then get interesting when we cross the rift.

The second advantage is we can keep the physics reasonable. I can't even begin to imagine how to compently describe a fight between the Jovian Confederacy and the Terran Dominion in hard sf terms. Trying to control the high orbials of a planets see siege, troop landings, the question of whether it would even be troops or drones.... Hard to imagine it not seeming like Georgian scifi writers imagining an invasion of the moon by manned cannon shells deploying troops in breathing suits to fight with laser muskets in infantry square formations.

Frankly, I don't think we can even begin to describe what a proper warship bridge would look like in a hundred years on Earth and what a starship bridge would look like, anything we guess at is going to be laughably wrong. At least a rifts warship is something pretty close to our own technology save for the antigrav and special fields for transiting the rifts. And since the ships aren't high performance FTL beasts, combat can be close ranged and brutal in the void.

The added bonus in the rifts is that there are many examples of other dead civilizations in traversed planets and they only get a little more advanced than where our heroes are now. Do they destroy themselves or are they destroyed by someone else? The means of destruction is sometimes nuclear, sometimes biological. Would put a bit of a damper in advanced research.


Man these templates are even screwing up on mobile devices. First time I've ever seen a text box refuse input. Maybe all these civilizations fail not when they build Cylons but when they create google and it turns evil.

jollyreaper said...

One other thought. What it would take to change today's missile dominance in combat would be superior defenses.

If we could build proper lasers that can zap a ballistic or cruise missile in a second, it could raise the required number of weapons for probable kill to an unaffordable level. Kind of kinetic star vs laser star on the high seas.

If rail guns work the way the navy hopes, a destroyer could hit with the force of a two million dollar cruise missile at 200 miles for maybe 10k a shot, cost of round and fuel for the magnets. Very difficult to intercept. But if the lasers do become good enough to take out rail gun shots then naval battles would see destroyers shooting anything over the horizon that doesn't have a defense laser and laser dueling with anyone who does have one the moment the laser mast pokes over the horizon.

Could provide a new impetus for building subs. Either way, removes the huge advantage held by planes and missiles. Removes the utility of civilian hulls with military launchers.

Anonymous said...

Byron said:"Milo:
I expect never to see someone shooting out of an airlock. A human isn't very good at precise shooting, when all is said and done. If it does happen, it will only happen once."

Umm, if it ever does happen, you WON'T see the person shooting at you; and it only needs to be successful once; the senario does seem like an act of desperation. People do desperate things all the time; it is surprising how offen these desperate acts succede.

On another matter, it seems that cargo ships would make excellent makeshift warships; payload bays and docking clamps for modules, long endurance engines and (possibly) life support for any crew. It seems to me that the first space warships would be modifications of existing spacecraft; or single-purpose, single-use craft built for a very specific purpose aginst a specific opponent (say, for example, Chinese ASATS vs U.S. sattilites, or American ABMs vs. N.Korean missiles) The first "Space War" will most likely be very ad hoc...

Ferrell

Anonymous said...

=Milo=



Francisdrake:

"Given the astonishing speed how fast electronics evolve, there seems little need for future manned explorers within our solar system, where mission control is never farther than a few light-hours away."

A few light-hours is still a few light-hours, and makes it difficult to coordinate fine movements or respond to new development. Everything you do has to be done very slowly and carefully so if something unexpected happens, you have hours to respond to it.

Additionally robots are simply less versatile than human hands.

I expect that robots will be used to scout out most of the solar system, and then humans will be sent to examine what robots have identified as the most interesting sites. Barring FTL-without-ansibles, humans will definitely not be sent out into uncharted wilderness - any human explorers will know exactly where they're going and what they're looking for.


"Can a man in the airlock with a gun in his hands damage a nearby vessel, without the gun seizing up due to lack of cooling?"

A small problem is the fact that typical spacecraft speeds are larger than the muzzle velocities of most chemical guns.



Jollyreaper:

"Oh this is even worse. The comments are threaded now? I'll have to scan up and down the whole bleeding comments section to find the new ones."

They don't look threaded to me. You have an account, so have you tried checking in your settings?

Incidentally how are comments supposed to be threaded given Blogger doesn't actually have a reply function and there's no way for their software to know which post you were replying to?

"Have to scan up and down the whole bleeding comments section to find the new ones" is a pretty accurate description of what happens when a comment is rescued from spam jail, though.


"Damn you, Google!"

On another note, Blogger also does and always has had trouble counting the number of posts in a thread. It seems every time a post of mine gets rescued from spam jail it ups the thread's post count by two rather than one, and recently some threads' post counts have gone up for no apparant reason since the last post hasn't changed (again, it doesn't look like threading to me).

And that's just the stuff Google is doing wrong on Blogger.

I'd say at least Google knows how to make a good search engine, except I'm getting fed up of their disregard for privacy.



Byron:

"Milo:
I expect never to see someone shooting out of an airlock. A human isn't very good at precise shooting, when all is said and done. If it does happen, it will only happen once."


That wasn't me suggesting that.


"I guess my view of the Kinzi lesson is that it's sort of like a bayonet. I don't want to get stabbed by one, but it only works at melee range, and it's not significantly more lethal then normal weapons."

The real question should not be how effective is a completely unmodified engine when used as a weapon, but rather should be how difficult it is to reconfigure an existing engine to function as a weapon using existing engine-servicing facilities.

fizz said...

This kind of scenario does strongly resemble "The African Queen" movie.

Apart colony on colony warfare, you could see jury-rigged military operations on the far fringes of a more traditional war.
For example, if you had a big war going on between Eurasia and Panamerica, the lone Callisto Eurasian scientific research station could have some heroic low-tech engagement to try disable the Panamerican gunboat guarding Ganymede refinery...

francisdrake said...

Byron:
You are right, looks like the 1/r² rule is really not in favor of the Kzinti Lesson. Even a 1 GW drive gives pretty low values at 10 km distance. Pointing the drive at the enemy might be ok to keep boarding parties away, but is much too low for a long-distance space fight.

Concerning the guy in the open airlock, shooting his gun at the approaching enemy: Definitely a desperate, last-ditch attempt. But might look good in a movie, with the soundless muzzle flashes illuminating the airlock interior ...

Byron said...

Ferrell:
Umm, if it ever does happen, you WON'T see the person shooting at you; and it only needs to be successful once; the senario does seem like an act of desperation. People do desperate things all the time; it is surprising how offen these desperate acts succede.
I just don't think this would succeed. Nobody is going to practice target-shooting against a moving spacecraft in zero-G. And the effective range of a Kinzi drive attack is longer then what a human could probably hit at. Not to mention the question of doing damage. A bullet is quite similar to a large micrometeorite.

On another matter, it seems that cargo ships would make excellent makeshift warships; payload bays and docking clamps for modules, long endurance engines and (possibly) life support for any crew. It seems to me that the first space warships would be modifications of existing spacecraft; or single-purpose, single-use craft built for a very specific purpose aginst a specific opponent (say, for example, Chinese ASATS vs U.S. sattilites, or American ABMs vs. N.Korean missiles) The first "Space War" will most likely be very ad hoc...
Absolutely. The following are a couple of post I made about a year ago, during Space Warfare XIII. As they're buried on page 4, I decided to post them here as well:
I've come to the conclusion that space warforces will develop along multiple paralell lines. Here's an example:
In the year 2283 there are two superpowers on Earth (the US and China) and a space colonies on Mars, Callisto and Ganymeade. The US and China have OPVs (Orbital Patrol Vehicles) to inspect the traffic in Earth orbit. The OPVs are small craft, with a crew of half a dozen and a dozen-man boarding party, along with a few small lasers. They're chemfuel, and operate from orbital bases.
Callisto and Ganymeade get into a feud over voliatle mining on Europa. Both have mines there, and Callisto says that Ganymeade is taking more then they are allowed to. Eventually, they mount weapons on a transport, and stop a Ganymeadean ship for inspection. The response is small, chemfuel craft carried as riders on transports. They're manned, as automation is hard to set up quickly. Space fighters. (Yes. I know it's heresy. However, it makes sense as a jury-rigged solution.)
Callisto responds with the same. Soon the situation escalates, with both sides converting a few transports full-time. Fighters remain the primary warcraft because neither side can afford that many full-time tranport conversions.
The US and China get annoyed at this, and decide to do something about it. The Jovian McGuffinite is too valuble to allow the supply to be threatened. Not having anything better, they cobble together a carrier and attach a bunch of OPVs to it. When it gets to Jupiter, there are two problems.
1. Neither side wants the crisis solved by Earth, at least the way Earth wants it solved. The carrier has no local supplies, sharply limiting it's use.
2. The Jovian fighters are more powerful than the OPVs.

Eventually the crisis is solved somehow. Ganymeade and Callisto remain at odds. Each body set (Earth, Mars, Ganymeade and Callisto) develops it's own set of space forces. Earth keeps the OPVs for orbital patrol. It also develops a deep-space fleet. Classical laser/kinetistars, etc. Ganymeade and Callisto develop nuke-thermal and chemfuel craft better suited for warfare around Jupiter. They're more worried about each other than Earth. Mars looks at what had happened and decides that they don't want Earth meddling in their affairs. They build a fleet that's similar to Earth's but lacks much deep-space capability. It's more kinetic-heavy as well.
Each has a seperate, paralell fleet because they had different starting points, but all of them work.

Byron said...

(split due to length)
Slightly later:
The more I think about it, the more I become convinced that manned warcraft, and even manned fighters, will have their day. That day will be brief, but it will happen.
The period I refer to is the first war in any given sphere, excepting Earth. I use the term sphere to mean any area that involves multiple powers. I'll use an earlier example. Europa and Callisto are in conflict over access to the output of the Rare Earth mine of Metis. Callisto mounts a laser on a freighter and forcibly inspects Europa's freighters. Europa responds by putting a missile rack, a chemfuel rocket, and a hab module in a cargo module, and carrying it with their freighters. Some freighters are also modified. All are manned.
Why? Simple. Setting up a secure laser comm setup isn't impossible, or even that difficult, but it does take time. I'll make a SWAG, and say 6 months minimum with a lot of money. I doubt that a space colony will have the money to afford it while fighting a war. With more time, the cost drops dramatically.
This would be brief, just like many eras we romanticize, but I think it would happen.


francisdrake:
Concerning the guy in the open airlock, shooting his gun at the approaching enemy: Definitely a desperate, last-ditch attempt. But might look good in a movie, with the soundless muzzle flashes illuminating the airlock interior ...
Been done. Firefly, Our Mrs. Reynolds.

Byron said...

francisdrake (again):
You are right, looks like the 1/r² rule is really not in favor of the Kzinti Lesson. Even a 1 GW drive gives pretty low values at 10 km distance. Pointing the drive at the enemy might be ok to keep boarding parties away, but is much too low for a long-distance space fight.
Absolutely. It's sort of like ramming at sea today. That does give me an idea, though. Ramming does occur, but usually because it's an accident instead of an act of war. The same could apply to blasting someone with the drive.

Thucydides said...

Remembering the story, the only reason the Kzinti lesson worked at all was the Earth ship's exhaust was in a tightly focused beam, essentially turning it into a huge laser weapon.

For most normal types of drive, even highly energetic fusion drives with tightly focused magnetic nozzles, this really doesn't seem to be possible at all. IF anything, I'd be more worried about the engineering crew jury rigging the magnetic nozzle for the fusion drive to act as a massive coil gun to launch metal objects as a sort of high tech shotgun.

I think Jerry Pournelle had the right idea in his "Those pesky belters and their torchships", where he pointed out that if this sort of drive existed, there was effectively a unified Solar System; Earth bureaucrats could show up at will to enforce their directives anywhere in the asteroid belt and beyond. Given the demographic, industrial and economic advantage Earth will have overall (even if individual Island 3 city states have a greater per capita GDP), then space war will probably resemble guerrilla war on the ground and mine warfare in orbital space in any contest between the Earth and colonies.

One thing which no one has mentioned yet is the colonies might find it more effective to conduct cyber war and hack each other's systems. Since space is such a dangerous environment, there might be some tacit agreements to limit hacking to disrupting industrial systems or reducing the efficiency of the banking system, but life support systems are off limits (and isolated from the cloud to ensure no accidents happen either).

Byron said...

Remembering the story, the only reason the Kzinti lesson worked at all was the Earth ship's exhaust was in a tightly focused beam, essentially turning it into a huge laser weapon.
I think that thermo might produce problems with that. I know it might raise issues with focusing the beam tightly enough.

For most normal types of d rive, even highly energetic fusion drives with tightly focused magnetic nozzles, this really doesn't seem to be possible at all. IF anything, I'd be more worried about the engineering crew jury rigging the magnetic nozzle for the fusion drive to act as a massive coil gun to launch metal objects as a sort of high tech shotgun.
I have serious doubts about that, too. Just because big magnets are involved doesn't mean that a coilgun is practical.

Earth bureaucrats could show up at will to enforce their directives anywhere in the asteroid belt and beyond.
Unless the ship runs into something. Which is the issue with running around at high velocities.

Byron said...

I think that some of the supposed threading is actually Milo's stuff being let out of spam jail. The rest is due to the use of mobile mode, which does have a reply setting.

Milo:
That wasn't me suggesting that.
Sorry, it was francisdrake. My bad.

The real question should not be how effective is a completely unmodified engine when used as a weapon, but rather should be how difficult it is to reconfigure an existing engine to function as a weapon using existing engine-servicing facilities.
Again, I'm not sure. Exhaust will be as tightly focused as reasonably possible, no matter what. Making it more tightly focused to use as a weapon is probably impractical. Not to mention that hitting anyone with it is a pain in the rear.

Cheery Reaper said...

My thoughts on largely automated ships or using waves of drones are this:

You can handwave it with something about how advanced hacking programs are to super-advanced computers what the machine gun was to massed cavalry, after you wouldn't use a drone if it can simply be hacked and made to fire on your position before you even launch it. Thus giving you a reason to have large human crews on starships and human pilots for your Lancers.


And, most of all, because no one wants to read about the life and times of a missile bus.

We don't read the story of Arthur's knights to see what life's like for the knight's trusty steeds, we want to read about Gawain dominating at the tourney and being a badass.

jollyreaper said...

Ah. The threading is due to Blogger using different templates. There are reply buttons after each post in the mobile interface but not in the desktop version. UGH.

I'm not sure what's the bigger mistake, everything Google's doing across all their web properties or Windows 8 and Metro.

Anonymous said...

Well, if there are wars in space, it would either be two or more powers fighting a general war with a space theater of operations, or a limited war between two or more powers fighting over a very specific issue; or, (if transit times make quick intervention impractical)off-world outposts/colonies have a dispute that escalates into shooting, where they cobble together whatever to bash the other guys...

Ferrell

Geoffrey S H said...

@Byron:

"NO!!!!!!!!
Why did someone have to bring up zeppelins? They have antigravity, so there's no need for them."

You may not always want a platform that uses anti-gravity. Reconnaisence zeppelins might be quite useful without the complicated anti-grav tech installed.... assuiming the zeppelins themselves aren't an intrinsicly complicated craft which in the real world they are right now. Nevertheless, there might be a niche use for such craft.

Anonymous said...

=Milo=



Cheery Reaper:

"You can handwave it with something about how advanced hacking programs are to super-advanced computers what the machine gun was to massed cavalry, after you wouldn't use a drone if it can simply be hacked and made to fire on your position before you even launch it."

Unlikely. Computer hacking is essentially a sucker punch - it works by exploiting a weakness the victim wasn't aware of or didn't take sufficient steps to protect against. Once any given software exploit becomes known, fixing that exploit is going to be far easier than remodelling all your ships on a hardware level. As such, any computer hacking that does get done (against a "serious" target like a military, anyway) is going to be done by humans (or strong AIs) constantly devising new techniques, not a plug-and-play hacking program.

Far easier than actually hacking a program and making it do exactly what you want, would be confusing a program with decoy sensor input to the point that it doesn't know what it should be doing. It probably won't start shooting at its allies, but it won't do a very good job of shooting at its enemies, either.


"We don't read the story of Arthur's knights to see what life's like for the knight's trusty steeds, we want to read about Gawain dominating at the tourney and being a badass."

Horses have more personality than spaceships.



Jollyreaper:

"Ah. The threading is due to Blogger using different templates. There are reply buttons after each post in the mobile interface but not in the desktop version. UGH."

Why would they have a feature in the mobile version that isn't in the normal version? Isn't it usually the other way around due to more limited capabilities of mobiles?

Here's a horrifying thought: what if they're putting it in the mobile version as a test run while preparing to sic it on the rest of the unsuspecting userbase later? *shudder*


"I'm not sure what's the bigger mistake, everything Google's doing across all their web properties or Windows 8 and Metro."

These are times I feel happy having a decades-old computer that's on the verge of falling apart any moment. At least I don't have to put up with misfeatures targeted at the lowest common denominator of computer users.

Brett said...

@Cheery Reaper
You can handwave it with something about how advanced hacking programs are to super-advanced computers what the machine gun was to massed cavalry, after you wouldn't use a drone if it can simply be hacked and made to fire on your position before you even launch it. Thus giving you a reason to have large human crews on starships and human pilots for your Lancers.

The Battlestar Galactica solution? I don't think that would really work in space. For one thing, if it does become common, the drone-makers will start putting in things to blunt attempts to connect to the drone's computers - or better yet, make the drone's software run only on non-rewritable ROM chips. You'd have to do extensive testing beforehand to work out bugs, but that's going to happen anyways for stuff that expensive and complex.

In fact, I'm not even sure you'd have to go that far. If your drones are in the light-minute range or more away from the command ship, you can't really remotely control them in real-time. Just give them their instructions, and design them so they don't auto-run any signals sent at them.

And, most of all, because no one wants to read about the life and times of a missile bus.

I'm not convinced of that. You could make a tense battle going back-and-forth between the humans in the command ship monitoring the drones, and the drones themselves as they enter battle.

It would be more original, at least. Something different instead of Retro Space Opera #289, with all its tropes that were born in the SF Era before they had any clue of how good computers would become.

Byron said...

We discussed drones quite extensively in Space Warfare XIII. Competent design can remove the hacking problem without too much trouble.
Zeppelins do not make good recon craft. They're big, slow, and have a low tolerance for bad weather.

Byron said...

Sorry about the terse reply earlier. I was using my phone.
There are a lot of techniques to prevent hacking. One is to use directional lasers, and set up the drone to only accept lasers from the objects it's tracking as authorized control ships. Also, the enemy doesn't know the frequency of the lasers, and any signal that doesn't match is ignored (and I'm assuming solid-state or FEL, so the selection is very large). And don't leave open ports towards the enemy.
All in all, the risk is very manageable.

All-weather performance is vital is a scout vehicle. On a new world, you'll have no idea what the weather will do, and that could spell trouble for the zeppelin. Also, speed is good. Airships are slow. Unless antigravity has some major drawback, we won't see airships.

Cheery Reaper said...

Brett said: "I'm not convinced of that. You could make a tense battle going back-and-forth between the humans in the command ship monitoring the drones, and the drones themselves as they enter battle."

I'm not seeing that. You type in the command (if we still type in the future) and then send waves of mindless drones out and send back reports of how many you lost and which ones can be salvaged. It would be like reading an about an account stapling the red form to the blue form. It would be like reading about a day in the life of a screwdriver.

After reading this blog and reading Atomic Rockets my feelings on the matter are this:

Screw hard sci-fi. I'm going to write about my human piloted Lancers, galaxy spanning empires, vast swathes of unexplored space, FTL systems and energy shields. I don't mean that statement in a terse or impolite manner and I'm certainly not insulting any of the people here, but as a writer I have no fun in writing hard sci-fi and I never had much fun reading it.

Brett said...

@Cheery Reaper
I'm not seeing that. You type in the command (if we still type in the future) and then send waves of mindless drones out and send back reports of how many you lost and which ones can be salvaged. It would be like reading an about an account stapling the red form to the blue form. It would be like reading about a day in the life of a screwdriver.

I think it would be very easy. You show a wave of drones drifting in towards the targets. They're silent, almost invisible to visible light, shadows moving across the deeps of space.

Cut to the command ship, where you have a handful of people watching Augmented Reality displays showing the latest update on formation, as well as the target (it doesn't have to be a perfectly realistic display either). We hear someone do a countdown: "5 . . 4. . 3. . 2 *shifts to the view on the drones* 1 . . ".

Then suddenly, the darkness blooms with light. Laser fire is exchanged, beams invisible except in one case where the camera pans along to the viewpoint of the emitter, and ships burn. The drones' engines flare, over and over again as they maneuver. Missiles are launched, and we follow them as they fly towards their targets. Most are shot down (and you could have a really cool-looking sequence where a laser point defense system shoots down a ton of missiles), but a few get through to explode their nuclear warheads, and we see how the defense platform's electronics are ravaged by the burst of neutrons and radiation.

The whole thing is over in less than five minutes.

Sorry, I'm not much a writer. But you get the idea. It would almost be kind of like doing a submarine movie.


After reading this blog and reading Atomic Rockets my feelings on the matter are this:

Screw hard sci-fi. I'm going to write about my human piloted Lancers, galaxy spanning empires, vast swathes of unexplored space, FTL systems and energy shields. I don't mean that statement in a terse or impolite manner and I'm certainly not insulting any of the people here, but as a writer I have no fun in writing hard sci-fi and I never had much fun reading it.


Feel free, but you're writing in a saturated field. Hard SF is more difficult to write, but the limits it imposes can also make your writing more creative. What you described above is Star Wars. Star Wars was and is great, but it's also been done - and most of the imitations I've seen of it aren't that great. You can only rip off Flash Gordon and World War 2 combat for so long before it starts looking really bland.

Cheery Reaper said...

Brett said: "Then suddenly, the darkness blooms with light. Laser fire is exchanged, beams invisible except in one case where the camera pans along to the viewpoint of the emitter, and ships burn. The drones' engines flare, over and over again as they maneuver. Missiles are launched, and we follow them as they fly towards their targets. Most are shot down (and you could have a really cool-looking sequence where a laser point defense system shoots down a ton of missiles), but a few get through to explode their nuclear warheads, and we see how the defense platform's electronics are ravaged by the burst of neutrons and radiation."

That's a cool idea (and I'd say you do a nice job of describing things if you feel like writing) But the thing is I don't want to read about drones or hardware, I want to read about people, just like when I watch Blackhawk Down, I'm not watching the movie for the helicopters, I'm watching it for the story about the people.


"Feel free, but you're writing in a saturated field. Hard SF is more difficult to write, but the limits it imposes can also make your writing more creative. What you described above is Star Wars. Star Wars was and is great, but it's also been done - and most of the imitations I've seen of it aren't that great. You can only rip off Flash Gordon and World War 2 combat for so long before it starts looking really bland."

*Shrugs* just because I have blades and horses in a fantasy story doesn't mean I'm ripping off LOTR. Same with sci-fi, just because my story has FTL and whatnot doesn't make it Star Wars I'm not going to let Star Wars define what I write and I'm certainly not going to rip it off, because I hate rip-offs and I don't want to write something like that, not my thing.

Don't get me wrong I get where you're coming from and I'm not trying to one-up you or anything, but I also think that it can be just as challenging to write something original.

Byron said...

The plausibility and readability of hard sci-fi is another interesting topic. For a well-written example, try John Lumpkin's Through Struggle, The Stars.
Personally, I have more fun trying to figure out what we'll have then writing a story around it. Then again, I'm an engineer. It takes all types.

Brett said...

@Cheery Reaper
That's a cool idea (and I'd say you do a nice job of describing things if you feel like writing) But the thing is I don't want to read about drones or hardware, I want to read about people, just like when I watch Blackhawk Down, I'm not watching the movie for the helicopters, I'm watching it for the story about the people.

Even in Space Opera stories where the battles are a big deal, the battles themselves only take up a small part of the overall book/movie. You can easily have a story about "the people" that includes what I wrote above, rather than doing World War 2 dogfights in space.

*Shrugs* just because I have blades and horses in a fantasy story doesn't mean I'm ripping off LOTR. Same with sci-fi, just because my story has FTL and whatnot doesn't make it Star Wars I'm not going to let Star Wars define what I write and I'm certainly not going to rip it off, because I hate rip-offs and I don't want to write something like that, not my thing.

I know, and let me clarify that I don't think it's wrong or lazy to write less-than-diamond-hard SF (assuming such a thing exists, which I doubt*). I do think, though, that adding elements of "hardness" to a soft SF story can make it more creative and original.

I mean, just think about how most Space Opera takes flying off a planet into space for granted. What if it never really becomes cheap and easy to go into space from an Earth-like planet? I posed that in the "space trader" comments thread a while back - if your FTL is a "low-energy trick", then it could be cheaper to send goods and people 25 light-years than 100 miles off an Earth-like planet. Crashing down on a planet would be a Huge Deal, because recovery is not easy or cheap.

And, of course, there's other fun stuff for space combat if your ships aren't running on Magic Fuel. Think of ships trying desperately to avoid getting hit by giant surface-to-space lasers, running dangerously close to over-heating or running out of fuel (which would leave them drifting in space). They don't even need to be Space Fighters to get that - just ships trying to avoid assailants.

* Even most "hard" SF includes elements that I'm dubious about in terms of practicality. Anti-matter powered "torch ships" and all manner of nanotechnology silliness are two examples.

@Byron
Personally, I have more fun trying to figure out what we'll have then writing a story around it. Then again, I'm an engineer. It takes all types.

Same here (although I'm not an engineer). I'm a world-building nerd.

Eth said...

@Brett :
Another point is that each side may not know exactly the hardware of the other. For example, the heroes know that the other side has missiles, but they don't know how long their forward shielding will resist to the lasers. Which means that they don't know how many of them their laser point defences will destroy.
A dramatic moment could be when the heroes discover that the enemy laserstars have twenty percent more range than estimated, ruining their carefully planned strategies.

I hope to write such a story in some time. It would follow the crew of the command ship, waiting to see if their calculations and estimations are correct, their mind games with the enemy commander, but also the 'grand strategy' level with the political decisions, and maybe a part about the scientists and engineers trying to come up with better hardware, and spies trying to find what the other side is working on.

In fact, a setting with nascent space warfare like discussed here would be particularly adapted, with the unstable political setting, the "poor man arm's race" where each side try develop improvised weapons, the people using said improvised weapons without being sure what the other side came up with this time...

Anonymous said...

I would like to offer this scene from the film Skyline of Reaper and X-47B UCAVs attacking an alien spacecraft as an example of how drone combat can be made exciting:

http://www.youtube.com/watch?v=TNTD1j8YRuM

R.C.

Geoffrey S H said...

"Then suddenly, the darkness blooms with light. Laser fire is exchanged, beams invisible except in one case where the camera pans along to the viewpoint of the emitter, and ships burn."

Or the camera switches into infrared...

jollyreaper said...

@dread zeppelins in space

They really wouldn't work for the Voids and Rifts setting. No real use for lighter than air vessels but plenty of use for skyships which is what the antigravity warships would be.

I've done some thinking about the way the warships would be built. Nuke reactors for power, antigrav rings for hovering. Asymmetric repulsion can move them in a hover but for real speed they need to fire up the atomic turbines. They'd be powered directly by the reactors. Not open cycle engines like Project Pluto, the nuclear ramjet. These things won't be spewing radiation everywhere. I don't know what plausible propulsion numbers would be but I'd be happy with 70mph at sea level. It could well be less.

There would be aerodynamic control surfaces for use when at speed but when in a hover the reaction control system used in space can also be used in atmosphere.

I'm not sure about the optimal layout for the guns. I'm keeping the high energy physics low so there won't be lasers and ginsu beams. I'm thinking that practical concerns would keep the turrets smaller since you need to have good coverage. A naval battleship only has to worry about ship-sized targets on a two-dimensional plane and airborne targets over a 180 degree arc. A skyship could face either threat from 360 degrees. So I'm thinking the ship-killing guns would need to have the same range of rotation as your typical anti-aircraft mount.

Really big guns would be spinal mounts and require the whole ship to turn to aim. This might seem similar to the tank vs. tank destroyer discussion. Many tank destroyers lacked turrets, building the gun directly into the hull. Lower profile, cheaper to build, cheaper to maintain, less flexible in use than a tank. Not quite the same issue for a skyship. In the void the ship can maintain motion along a vector while turning to bring the gun to bear. In an atmosphere drag would slow it down but it might still be considered worthwhile. If the spinal gun is some ridiculous railgun that can fire shots that will hit 200 miles away, it might be useful for bombardment. The whole ship would have to rear back to assume a firing position.

Another thought. I always thought the most badass incarnation of the battleship we never saw would be one that's nuke-powered and firing nuke shells. The Army had a nuclear howitzer. Imagine a nuclear battleship giving a 9 gun broadside of a-bomb shells.

We decided to abandon the battleships after WWII because guided missile destroyers and cruisers could hit hard with nuke-tipped missiles and it was considered too difficult to armor sufficiently against nuclear hits.

If the naval planners of this setting see things differently, then perhaps bulk will be seen as protection. The Operation Crossroads nuke tests were pretty interesting. The ships didn't get that hot from the air burst, it was the underwater test that made them lethal. The heavier ships rode through the air burst. Given that the skyships have to be airtight to survive the void, it should be possible to keep the crews inside safe. There's no danger of sinking in the void and in atmosphere the primary concern is losing power and thus antigrav. They could sink if they lose power over water but could settle on land just fine.

Given that they're airtight I figure that water landings would be a routine thing. If they're not in motion they're going to be on water or the ground to save energy. Bodies of water are convenient because there's nothing to crush.

jollyreaper said...

I would concur with Lumpkin's book being very good. The ground combat scenes were a prime case of my own second-guessing.

The infantry combat was pretty much Next Sunday, AD. Guns, body armor, people shooting eat each other. What was missing? The stuff that's possible to imagine in the next 20 years, let alone what we could invent in the 200 years in the future the story takes place.

The most oddball but indisputably useful weapon I can think of is the tackler bot. It looks like an ambulatory plus sign, basically four tentacles joined at the base. For slow movement it walks on all fours, fast movement is a carthweel motion. What does it do? It tackles people and immobilizes them, using the tentacles to restrain limbs gently but firmly.

Got gunmen mixed in with civilians? Send in the tackler bots. You don't care if the bots get shot, they can tell baddies from civvies and even if they get it wrong, they're tackling, not killing.

There's also the idea of sniperbots. Mount a sniper gun on a quad rotor, have it hover a few hundred feet up over the battlefield. The targets might not even see it but it's scanning and highlighting options for the commander. He's flagged for kill/no kill on everything on the screen. If he's not sure of the target, some expendable drone could be sent in to confirm.

Hunter-seeker bots could be the size of rats or pigeons and go rooting through the terrain looking for targets. They could either have a gun or nerve agent darts. Hell, imagine a bug bot carrying a stinger full of VX. They say a drop of that on the skin can kill you in minutes.

I think the drones have as much potential to reshape the battlefield as chemical explosives.

From a storytelling perspective, I imagine trying to talk to HG Wells about the future and the stories he could tell in our own time.

"You have city-smashing bombs. How can there even be war?"

World war, no. That would smash everything to bits. So we have proxy wars instead, everyone agreeing to keep it on the down-low and not trigger a global war.

"Ok. But you have weapons that can drop fire from the sky. Why are people even fighting on the ground?"

Because those weapons are expensive. And you can't hold the ground from the air.

"Ok. But you have these armored vehicles with giant guns. Why are there still soldiers carrying guns?"

jollyreaper said...

Because tanks can't go everywhere and a soldier can knock on a door and enter a house. A tank can only knock down a wall and roll through a house.

"So what you're saying is that the maximum possible force applied is greater than we can manage in my time but not every situation calls for that force."

Yes. A cop may have a gun on his hip but the optimal solution for most of the situations he finds himself in doesn't involve shooting people.

"But the implied threat is useful."

Of course. And consider that a secret agent might work for a power that has ICBM's and supersonic bombers but he might only have a pistol on him at the time.

The real kicker here -- we don't see drug barons operating jet fighters because jets are damned expensive. We don't even see rival gangs in Mexico using Mad Max improvised combat vehicles. It's not practical, it's not useful. They stick with guns, explosives and knives. All of those are readily available, concealable, and useful. Cheap. We haven't seen a whole lot of IED's and I guess in part that's because they don't need that kind of weapon. The Iraqis are attacking armored vehicles. If we didn't have them they wouldn't need to go to such elaborate efforts. Hit jobs against people in civilian vehicles usually involve motorcycle gunmen. Less complicated.

I think improvised civilian toys are going to be big in the future. We've got RC quad rotors that you can fly from your cell phone. Strap on a bomb and you have a poor man's cruise missile. And it's cheap.

I would be very surprised if we don't end up seeing vast changes along these lines in the future.

Byron said...

Jollyreaper:
I think 70 mph is a bit low. For an antigravity vehicle, mass will have nothing to do with top speed in an atmosphere. The only thing that matters is thrust. 70 mph would be if you had similar thrust and aerodynamics to an airship. I'd imagine you'd have significantly more thrust per area, and similar (or maybe a little worse) aerodynamics. 150 mph would be low-end.

Another thought. I always thought the most badass incarnation of the battleship we never saw would be one that's nuke-powered and firing nuke shells. The Army had a nuclear howitzer. Imagine a nuclear battleship giving a 9 gun broadside of a-bomb shells.
They actually made nuclear shells for the Iowas. They were referred to as Katies (kTs).

Must we do surface warfare again? That was sort of fun, but I just don't have the time for it. :-)

jollyreaper said...

I'm not sure what a realistic upper speed would be.

With nukes there wouldn't be the immediate concern of fuel consumption. The battleships would have to strike some balance between aerodynamics and performing the mission. With subs and airplanes, everything is sacrificed for streamlining. With cars and trucks, there are some concessions but only to a certain extent. The gap between what we have in production and theoretically max efficiency designs is pretty wide.

If you use a big enough kick even a brick can go supersonic. We've proven that with the shuttle and F-4 Phantom. Is there a sweet spot between speed and mass of drive equipment? A supersonic flying battleship might have to devote a ridiculous amount of mass to reactors and engines.

There's also the question of whether or not there's any advantage to flying higher for lower drag and faster transit times versus the advantage of staying low and in the weeds to evade detection.

I don't even know where to begin for back of the envelope calculations on what it would take. The mass of the battleship would remain even with antigrav on so you could treat it like maglev.

One other thought: the rockets used in space are fueled by material taken from a planet. I don't know if it makes more sense to load water, crack it into liquid oxygen and hydrogen. Whatever the method, I want the ships to have their own ability to make fuel once they're back in atmo. But those engines, they'd have a hell of a kick and would be pretty impressive if lit off in the atmosphere. I wonder what kind of performance could be have if they pointed the bow up at the sky and fired off the mains, 105% throttle, and burned off everything they had in the tanks. Could you get the ship up in a ballistic arc 20 miles in the air and coming back minutes later hundreds and hundreds of miles away?

It wouldn't have the same fuel/mass ratio as a space rocket. Wouldn't need it, either. There's not a whole lot of maneuvering to be done in the void, just starting and ending burns. Still, carrying a full load in the tanks would be a significant mass penalty for atmospheric flight. When to start tanking up for hitting the void and how long it takes to do so would all be factors for the captain to keep in mind.

Anonymous said...

Jollyreaper,

Frankly, I don't think we can even begin to describe what a proper warship bridge would look like in a hundred years on Earth and what a starship bridge would look like, anything we guess at is going to be laughably wrong.
========================

I think we can describe it just fine .. we just cant describe it and still obey the Zeroth Law.

Most likely the bridge will be several hundred thousand lines of code running on one of the 3d Electronic IC/ Quantum computer concepts.

Human craft will not be the ones fighting .. for all the same reasons that purpose built warships will dominate over generalists.

Sure there could be exceptions .. you could have a manned exploration /science /combat ship .. that costs 10X as much as a comparible space purpose built warship. But you wont use those for going to war.

(SA Phil)

Anonymous said...

(SA Phil)

As to the hacking "problem" of drones ... if you really had that vulnerability you would just shut off the Drone's "reciever" before it got close enough to the enemy for them to hack it.

More likely though no one will be able to use your transmission paths anyway -- there are simply too many ways to vary your communication.

When we studied electronics communications in school we learned 10 different modulation types in 10 weeks ...

There is simply too much room for variation when you are sampling millions of times per second.

Assuming you even need to really communicate with the thing. Your war AI drone wont need to follow the three laws of robotics.

Byron said...

Jollyreaper:
I don't even know where to begin for back of the envelope calculations on what it would take. The mass of the battleship would remain even with antigrav on so you could treat it like maglev.
One of my classes right now is basically all about that. As for the mass, I said top speed. If you're in equilibrium level flight, then you don't care about mass. Thrust equals drag.
One thing to keep in mind is that there is no lift involved here. That dominates aircraft design.
Drag does increase in the supersonic region, but up to about Mach .7 it scales with the square of velocity. I'm not sure exactly what kind of engine you'll go with, but if you can give me harder numbers, I'd be willing to at least see what happens.

Rick said...

The rift setting may not have giant gas bags (except perhaps some human characters), but it certainly has airships.


The tension between plausibility and story-ability is really one of the key issues of this blog.

I'll take as a given, per Burnside's Zeroth Law, that a story is about people. Even if it has robotic characters, they are characters by virtue of having feelings, aspirations, yada yada. (I think there's a real question whether even high level AIs would actually have these traits - see What Do AIs Want?)

Back to the point, in principle there is no reason the story couldn't be about the crew of the command ship, which will certainly be targeted. It is the 'bridge' of the whole constellation.

For that matter, no one in a modern era naval battle fights in the sense that Miguel de Cervantes (who served at Lepanto) would have understood. They basically do industrial work of various kinds, while under attack.

The problem, in story terms, may be mainly that one-hit one-kill weapons don't generate much drama. You are fine, until suddenly you are dead.

Anonymous said...

Milo:



Jollyreaper:

"We don't even see rival gangs in Mexico using Mad Max improvised combat vehicles."

Those are used in other parts of the world, though.


"motorcycle gunmen"

Modern-day cavalry?



Rick:

"The problem, in story terms, may be mainly that one-hit one-kill weapons don't generate much drama. You are fine, until suddenly you are dead."

One-hit one-kill weapons can create a different sort of tension in stealth-based combat scenes, where each side tries to be the first to locate and shoot the other.

Oh right, but there's no stealth in space :)

This is why so many SF authors come up with force fields, which allow ships to take more hits ever from really powerful weaponry. And with stealth in space.

Force fields have even less scientific basis than FTL travel, though (at least ones that are able to stop stuff other than charged particles).

Eth said...

"The problem, in story terms, may be mainly that one-hit one-kill weapons don't generate much drama. You are fine, until suddenly you are dead."

The Hunt for Red October managed that quite well in the final submarine battle, with one-hit one-kill torpedoes. And despite being a 'war' movie, the drama is not all about battles themselves.
But maybe we should also draw comparisons with movies about chess champions. It's all about careful planning, outsmarting the other guy and manipulating and positioning one-hit one-kill units across the board/space; until one side can take the only important one, the king/command ship.

Anonymous said...

In modern warfare, the tension comes from avoiding being hit; the manouvers, the counter-measures and defenses, the electronic dueling, and the strategies used to put the enemy off-balance; what will work and what will get you killed? Will your attack kill the other guy before he kills you? I don't know about you, but that spells high emotion to me...
One-hit, one-kill...but only if it Does hit...

Ferrell

Anonymous said...

Oh, and another thought; wouldn't interstellar survey ships carry arms for the "what if"? You Are, after all, going into the unknown.

Ferrell

Chris Gerrib said...

A few thoughts:

1) I've done research on laser effectiveness. Shooting at thin-skinned (aluminum) missiles, the laser has VASTLY more range than any anti-missile system. I'm talking ten times the range.

2) If you're doing asteroid mining, you'll want powerful lasers for drilling. Anchoring conventional drill rigs will be problematic in a very-low G environment.

3) Improvised weapons will be the order of the day for any first-generation deep space war. See the history of air forces in WW1.

Byron said...

Chris:
Asteroid mining lasers are entirely different from laser weapons. They are probably fixed-focus, and have small lenses, as the target is very close. A laser weapon needs variable focus, a large mirror, and has to be able to pivot at least a little for fine adjustment. Not remotely the same.

Anonymous said...

=Milo=



Ferrel:

"Oh, and another thought; wouldn't interstellar survey ships carry arms for the "what if"? You Are, after all, going into the unknown."

If you're going somewhere no-one has ever been before, who's going to shoot at you?

Byron said...

Also, we would probably be able to detect any threat before the survey ship leaves. The chances of an enemy with FTL capability setting up shop in the time between when the last light leaves and you get there. I'm assuming, of course, that you're not going very far out. On the other hand, at least at first, that seems quite reasonable. And yes, we would detect almost anything above the level of a single survey ship.

Anonymous said...

=Milo=



Byron:

Well, not so clear-cut. The "no stealth in space" thing applies across interplanetary distances, but trying to point a telescope at a solar system many lightyears out is more tricky. Even the exhaust from a fleet of ships could be drowned out next to the glare of the star. And, well, you don't see anything until years after it happens.

jollyreaper said...

@ Byron

Skyship powerplant would be nuclear. Engines would either be directly nuclear or electric depending on the technical complexity.

http://en.wikipedia.org/wiki/Aircraft_Reactor_Experiment

Nuclear-powered bombers were seen as too risky by the Air Force. The potential engines mention using liquid metal reactors. I know that the Russians played with those on some of their subs. Lose power and the whole thing turns solid. You're not bricking your reactor, you're ingoting it.

If the flaming jets idea doesn't work, maybe they just use a ducted fan technique, machinery is protected deep within the hull of the ship but you see air intakes and exhausts.

Fighters would not be flying with reactors. Fighters would have a turbine running an electric generator to provide power to the antigrav when hovering and use aerodynamic lift when at speed. Some models might have the dedicated turbine for electric or they might use a thrust deflectors to route the exhaust gas past the generator. Maybe fighters operating like the Harrier that only need antigrav for take-off and landing would use a deflector and aircraft that operate more like choppers with a lot of hovering would have the dedicated turbine for power and then use the proper jets when traveling at speed. There's no spinning rotor to worry about which is why conventional choppers can't go supersonic, Airwolf aside.

All jets would burn hydrocarbon fuels. It'd be pretty cool if the ships could make their own avgas. Even the process is too expensive for use in the civilian world, it might make sense for shipboard use if trying to transport the fuel millions of miles from planet to planet is even more expensive.

For void operations, fighters would rely on fuel cells.

http://en.wikipedia.org/wiki/Algae_fuel

I'm assuming that hydrocarbons would be easier to work with than cryogenic fuels. I'm thinking cryo for the skyships themselves.

The Sea Dragon sea-launch heavy lifter proposal suggested the cryo fuels could be provided by a repurposed nucelar carrier serving as a support ship for the launch efforts.

http://en.wikipedia.org/wiki/Sea_Dragon_(rocket)

@Rick
They basically do industrial work of various kinds, while under attack.

The problem, in story terms, may be mainly that one-hit one-kill weapons don't generate much drama. You are fine, until suddenly you are dead.


Even WWII didn't have proper fights by Nelson's view. Strikes are launched over the horizon between fleets that don't even see each other (the great carrier battles), the surface gun battles are between ships that are specks to each other, the ships not closing to ranges where the crews could shout insults at each other, no boarding actions, etc. But WWII certainly had its own kind of tension. Submarine combat is hard to compare to anything else in warfare prior to it, maybe the darkness and claustrophobia of tunnel warfare mixed with drowning.

jollyreaper said...

Even if the only humans in the battle are the leaders in the bunker with missiles striking continents away, drones buzzing over the battlefield, there's a tension to be had. It may not be swinging axes and shooting pistols with cinematic flourish but there's tension to be had. Imagine the recent Hitler movie, the Downfall, with drones and terminator bots outside and them inside. The psychology of the situation is pretty much the same.

Come to think of it, the feel could be quite similar to a gambling movie. The gambler is playing against the odds. In pool there's some skill involved as well as luck but in something like poker it's luck and bluff. The leader in the bunker is placing bets and getting results.

Still, there's some potential for special effects spectacles even while the leaders are in the bunker.

Consider this surveillance drone flown by the protesters recently.

http://www.youtube.com/watch?v=o3OB_4BT1LA&feature=related

Now imagine that it's an invasion by an enemy's automated forces. Heavy bombardment is over. Strategic defenses have been smashed and now the ground forces are securing the city. The enemy is seeking economic control which means they need plant and equipment intact. Military bases, defense installations, those have been obliterated. Some cities have been obliterated. Economic targets are being suppressed and occupied. The leader bunker is in the middle of the capital and strategic weapons capable of cracking it would take out the rest of the capital. It needs taken relatively intact and it's also a propaganda coup to have the leaders captured, dead or alive. The defenders will have HD footage of the fighting.

If the Rebel Alliance used drones instead of manned fighters, the Battle of Yavin would certainly have had a different feel. Possibly not for the people in the CIC on the planet since they were facing destruction either way. But for the viewer there was the shift in perspective between the immediacy of the fight and the anxiety in the bunker listening to the fight on the radio trying to imagine what was going on.

PS email followups are finally back! Thank Cthulhu.

Anonymous said...

(SA Phil)

Once you reach a certain level of "AI" the drones wont need to be controlled by anyone.

And thus the only human drama would be the humans the attacking drones manage to kill after getting past the defending drones.

You could build a lot of drama into such a story -- but not a human military type of drama.


===========

As to the anti-grav discussion - wouldnt a lot depend on what powers the anti-grav? If you need a massive amount of electricity to power the anti-grav, you could use the heat from that powerplant to expand intake air -- and use that for your forward thrust.

Anti-grav seems operatic to me though. Even if you just did Today's Tech+Anti Grav.

Byron said...

Milo:
Well, not so clear-cut. The "no stealth in space" thing applies across interplanetary distances, but trying to point a telescope at a solar system many lightyears out is more tricky. Even the exhaust from a fleet of ships could be drowned out next to the glare of the star. And, well, you don't see anything until years after it happens.
Not exhaust, radio signals. And I know that light lag would be an issue. However, I also expect that the average expansion rate will be somewhat sublight. You'll know there's someone a few systems over, and either arm your ship, or take steps to avoid them. Also, the chances of a previously undetected race deciding to move in in the 10 years (or whatever) it's been since you checked is fairly small.

Jollyreaper:
How much power does the antigrav use? You might be able to power it the same way a modern jet does with a feed from the fan shaft.
I'm skeptical of make your own fuel. For hydrocarbons, you need feedstock, and sunlight. Yes, you can use the reactor, but you have to harvest the CO2 and water somewhere. The algae takes time to grow, and I don't want to see the mass numbers.
Also, seawater may not always be available. And it takes time to crack it out, which is not a good thing in an emergency.
I guess the question is what you're trying to do. If the vessel in question is a survey craft, then the above makes sense. You need to be able to sustain yourself away from base for a long time. However, if the vessel is instead a warship that operates with the fleet, then have the fleet train do it.
And yes, hydrocarbons are easier to handle then cryogenics. But what about metastable metallic hydrogen?
Nuclear direct drives are not things to use at low speed. Even the indirect ones would likely be frighteningly radioactive, and that sort of system works better at speed.

I'm really not qualified to comment on the story potential of drones.

Anonymous said...

(SA Phil)

I think the 'no stealth in space' idea also applies to STL invaders from other stars. There is so much energy involved in the insane length of acceleration involved that you could "see" the heat (etc) for many light years.

Eth said...

Unless the Q-thruster evoked here :
http://atomicrockets.posterous.com/nets-2012-abstract-highlights
is somehow the real thing. Not that I expect it to be, but one never knows.
You would also have cheap and almost invisible interstellar WMD, among other things.

Anonymous said...

The impression I got of the "Q-Thruster" was that it had the same degree of performance as any other ion or plasma thruster, just without the need for propellent (it still needs power to operate, it seems).

Repurposing a water tank or two, plug in a modest life support, a computer for control, whatever scavanged sensors, a chemfuel rocket and manovering system, and an improvised weapon(s) and your moon-based colonial millitia has an orbital gunship.

Ferrell

Anonymous said...

=Milo=



Not needing propellant is a huge thing. The concept sounds farfetched, but if it works, wow!

Stevo Darkly said...

I really like Jollyreapers "Voidrifter" setting, or whatever you'd want to call it. Especially with a bloody-minded, human-sacrificing Aztec-descended enemy. I'd buy those stories. :)

Ferrel said: "Oh, and another thought; wouldn't interstellar survey ships carry arms for the "what if"? You Are, after all, going into the unknown."

I agree. Good point.

Milo said: "If you're going somewhere no-one has ever been before, who's going to shoot at you?"

Ferrel did say "interstellar," and so your "no one" implicitly means "no one from our own civilization." There remains the possibility of hostile natives, or an aggressively expanding alien empire.

Stevo Darkly said...

Regarding Jollyreaper's anti-gravity skyships in flight within a planetary atmosphere:

Byron said: "One thing to keep in mind is that there is no lift involved here. That dominates aircraft design."

A half-quibble/suggestion: You could design your large skyships to be somewhat aerodynamic lifting-body shapes so that their hulls generate some lift during high-speed forward flight in an atmosphere. That could reduce some of the power you need for the anti-gravity lifters, which you could divert instead to your forward-thrusting engines, or something else.

I also like the idea that the skyships could land in bodies of water.

Personally, I'm envisioning a vessel that looks like a cross between an Aeroscraft (but no gasbags inside that hull), a giant Silver Dart spaceplane, and the Spaceship Yamato.

Byron said...

The Q-thruster:
I'm so incredibly skeptical on this one. "Virtual particles in the vacuum"? How is this not a violation of conservation of momentum? Until someone explains how it works, I wouldn't touch it.

Aerodynamic lift:
Lift has its own penalties. Using it will increase drag, slowing the ship. And lifting bodies are not terribly efficient on that front.
The big question is how much power the antigrav takes. If you can keep a major airshp aloft for two and a half weeks on a pair of D-cell batteries, forget it. If it takes a souped-up nuclear reactor to get it off the ground, then it might be worth a try.
If it is really power-intensive, then maybe fighters only have a short amount of time on it. They use capacitors to power it, and recharge them from the engines.

Landing in water is an interesting concept, but I wouldn't want to be the structural engineer.

jollyreaper said...

As to the anti-grav discussion - wouldnt a lot depend on what powers the anti-grav? If you need a massive amount of electricity to power the anti-grav, you could use the heat from that powerplant to expand intake air -- and use that for your forward thrust.

Anti-grav works because of mcguffinite found inside the void. There are clumps of rocky debris not unlike asteroids. Some clumps are small, some are world-like. This stuff is unlike the normal matter found on terrestrial worlds.

Mcguffinite-A is the stuff that works well for antigrav. Run electricity through it, you get a repulsive force. Mcguffinite-B is clean nuke fuel for running the reactors. Extant nuke reactors are too expensive to be cost effective. Even the military can barely afford them. Mcguffinite-B helps to lower the costs as well as allowing for nuke-scale explosions with much less fallout. So if the neutron bomb leaves buildings behind and kills with radiation, these leave the fallout behind and kill with huge effin' 'splosions. Woooo!

Anti-grav seems operatic to me though. Even if you just did Today's Tech+Anti Grav.

Completely operatic. Rather than trying to tell the story that fits the setting, I'm constructing the setting to tell the story I want.

Space combat will not be like WWII in space. WWII is not like sailing in the Trojan War. Armored warfare in tanks is not like mounting a charger for a cavalry charge.

The void setting tries to satisfy the needs of:
A) Interplanetary-scale warfare
B) Interplanetary empires and really alien cultures that are still just humans
C) Space combat that feels a bit more like Star Wars that can be enjoyed without feeling like your brains need scooped out and replaced with guacamole
D) Keep the tech close enough to the modern day in a plausible fashion so we don't have anachronistic schizo-tech.
E) Put a plausible decelerando barrier in place (cultural, technical) so that a relatively stable and static tech level for long periods of time doesn't strain credulity.
F) An economy and human motivations that still make sense, at least to me.

Once a species is able to operate at the interstellar level, resource competition seems like it should be over. You're post-scarcity. Need space? Go where you can get it. There's plenty for everyone. Nobody's fighting over the last chicken leg at an endless buffet. The only room for conflict at this point would be philisophical/doctrinal. We don't like what you're thinking or doing and will force you to stop.

I'm not saying that it can't be done but I find post-scarcity far-future stories really hard to come up with. The only certainty that I have is that it would be as far beyond a 2001 portrayal as 2001 is beyond the typical Star Wars space opera.

jollyreaper said...

How much power does the antigrav use? You might be able to power it the same way a modern jet does with a feed from the fan shaft.

Not sure. there's a mix of advantages for different vehicle types.

Fixed-wing aircraft that use lift surfaces only and require runways would be the cheapest form of flying transport.

Water ships would be the cheapest long-haul transport on worlds with a lot of water. If you're only moving stuff around on the single planet, this might be the way to go.

Skyships would be used to move goods between the worlds via the void. They might also be economical on planets where you don't want to have to put a lot of infrastructure on the ground.

If you're hauling bulk ore along a fixed route then maybe it's worth the effort to build a rail line.

All the variables for this setting are on little sliders that can be moved back and forth to create the most interesting mix of advantages and drawbacks.

You can't really have Pirates of the Caribbean without European powers sending lots of ships and manpower to the Americas to extract wealth. You need the wealth to draw imperial interest, to have high-power conflict. Otherwise you'd have Indians in dugout canoes raiding each other but not tall ships and dashing naval officers hunting dastardly pirates.

I'm skeptical of make your own fuel. For hydrocarbons, you need feedstock, and sunlight. Yes, you can use the reactor, but you have to harvest the CO2 and water somewhere. The algae takes time to grow, and I don't want to see the mass numbers.

The fighters need some kind of fuel. I'm not wedded to any particular solution, would just like to avoid as much handwavium as possible.

Also, seawater may not always be available. And it takes time to crack it out, which is not a good thing in an emergency.

Right. What happens if you clear the void and the planet you have to transit looks like Arizona? You might have air to compress, nothing more. That's why I was wondering if the rockets should count on burning two fuels or if it should be more a matter of compressing any kind of useful, available gas and blasting it through the reactor and spewing it out the other end.

jollyreaper said...

Given that I'd like to keep these ships as close to realistic as possible, the idea that they can carry all the fuel they need for big rockets in some small tanks doesn't work for me.

I guess the question is what you're trying to do. If the vessel in question is a survey craft, then the above makes sense. You need to be able to sustain yourself away from base for a long time. However, if the vessel is instead a warship that operates with the fleet, then have the fleet train do it.

The way I'm envisioning operations, the distances can become quite ridiculous. Yes, a given world will have fleet support facilities and can directly supply any fortifications kept on the other side of the rift in the void. But fleets undertaking long voyages could end up millions of miles away from support.

A long trip could involve leaving base for a two week trip through one void, a thousand mile transit on an empty, barely inhabitable planet, a week through another void, 10k mile trip across an iceball world, a month in the void, now crossing an inhabited world of an allied power and dropping off the goods.

There are some short runs through the void between known ports but there's always oddball rifts to find and really, really out of the way routes that might not make economic sense for ships but could prove invaluable for a military strike.

"Hey, it's about two weeks to get from our base to the enemy world. But if you take this really, really, really out of the way route, it takes about six months but you end up coming out in this planet that they didn't know has a back door. It's in their backyard and then you can come through an unfortified rift right into that world and they'd have little warning."


And yes, hydrocarbons are easier to handle then cryogenics. But what about metastable metallic hydrogen?
Nuclear direct drives are not things to use at low speed. Even the indirect ones would likely be frighteningly radioactive, and that sort of system works better at speed.


Dunno. Even the realistic stuff seems like handwavium to me. lol

Byron said...

Jollyreaper:
All in all, I find the setting to be a very interesting idea. Your requirements seem reasonable, and it meets them all.

My suggestion regarding logistics is to decouple the combat skyships and the support vessels, unless the vessel in question operates independently. This would give a fleet ship a clear advantage over a patrol one, but that's to be expected.

jollyreaper said...

@ Darkly

Lifting body ships

This is all something to be played with. There would be trade-offs to consider for aerodynamics versus the usefulness of a lumpy design. How big do the guns have to be to remain useful? Could that sort of turret recess in any meaningful way? How much superstructure is required? Search radar needs to be all around. Are emitters on the bottom required? Would everything be phased array at this point so appear as nondescript flat panels along the hull like modern Aegis? Seeing as the whole ship has to be able to seal up tight for combat anyway, would they by nature end up looking a lot more like subs (and thus spaceships) than flying battleships? (I'd think so but to what degree?)

A plausible spaceship could very well be built with floors perpendicular to the long axis(classic rocket ship layout) and the newer convention of floors parallel to the long axis (like any sea-faring vessel) would be implausible. Buuuuuuuut the voidships now have a rationale for that sort of layout since a large portion of their time would be spent in atmosphere under some form of gravity anyway.

Just had a flashback to Robotech. Alien super-dimensional fortress crashes to Earth, we rebuild it, aliens try to reclaim it. The aliens attack on launch day and the humans are trying to get up into space so they can fight back. Antigrav generators fire up and the ship is lifting from the launch cradle. There's some shuddering and machines tear up out of the hull. Oh, crap! Those are the antigrav pods! The ship crashes back down. Oops.

That could reduce some of the power you need for the anti-gravity lifters, which you could divert instead to your forward-thrusting engines, or something else.

This would really have to be run past some plausibility checkers. What you're suggesting doesn't sound unreasonable but I've got a gut reaction that maybe the smaller frigate types could benefit from that but the larger battleship types might be too bulky. But I have no idea even what a reasonable top speed would be.

The vague image in my mind is that the ships would be about as aerodynamic as a lumpy bullet, big turrets might not be useful so they'd be using smaller turrets with excellent traverse and elevation and fire nuke and high explosive shells to increase firepower.

I also like the idea that the skyships could land in bodies of water.

The ships wouldn't look anything like Space Battleship Yamato, though, since they aren't converted WWII battleships.

Also, seeing as they wouldn't need to be worried about conventional hydrodynamics or transiting canals, what other shapes could work? Would a flying saucer be reasonable? I always loved the Basestars from Battlestar Galactica but the motherships from V might be a little more streamlined.

jollyreaper said...

With power requirements, I don't know how it would scale. Like if we compare it with conventional tech, you can build a helicopter but a heli-carrier is impossible.

My gut reaction is that conventional power is sufficient to run antigrav for smaller vehicles and that's how the military would handle it for small craft. There might even be conventional airships incapable of going through the void that burn fossil fuels. The two strikes against them for void use would be a) no convenient source of air for burning bunker fuel with and b) they couldn't possibly carry enough fuel for the longer trips.

Maybe, just maybe super-specialized civilian ships could get away with a dedicated run where they're hauling mcguffinite from the void right next door. Burn diesel up until crossing the void, run on fuel cells on the other side and burn rockets to make it to the mine, burn rockets to make it back to the rift, switch to diesel on the other side.

As for gravity in the void, if it's reasonable to think that antigrav could work in reverse, flip the switch and now antigrav sucks the crew back down to the deck. If that seems like too much of a stretch, then smaller ships might be stuck with zero grav and larger ships might have spin sections. Spin up in the void, spin down before hitting atmo again.

The concept requires a lot of hammering on before it's ready for primetime.

As for the name of the setting, I like the old Black Sabbath song "Into the Void." ItV.

Anonymous said...

=Milo=



Stevo Darkly:

"Ferrel did say "interstellar", and so your "no one" implicitly means "no one from our own civilization". There remains the possibility of hostile natives, or an aggressively expanding alien empire."

Unlike Earth, though, we aren't expecting every nook and cranny of space to already be settled by natives.

Unless we actually know there to be hostile factions operating in that region of space, "just-in-case" defense doesn't seem like a high priority.



Jollyreaper:

"Once a species is able to operate at the interstellar level, resource competition seems like it should be over. You're post-scarcity. Need space? Go where you can get it. There's plenty for everyone."

Space is plenty. Space you can live in is more rare.

If terraforming a planet is difficult - which seems a guarantee under even the most optimistic assumptions - then there would be a logical advantage to just stealing one someone else has terraformed.

McGuffinite is harder to justify fighting over. If you need, say, helium-3, then there's bound to be plenty of gas giants in the galaxy that nobody has claimed yet, and the ones that have been claimed aren't going to run out anytime soon even if multiple people mine them. The exception is McGuffinite of biological origins - but what could that be?


"A plausible spaceship could very well be built with floors perpendicular to the long axis(classic rocket ship layout) and the newer convention of floors parallel to the long axis (like any sea-faring vessel) would be implausible."

If you don't mind being highly unaerodynamic, then a "flat" ship - with the engines laid out along the bottom side - would be easier to keep stable than a rocket shape. It would also give you a lot of space for engines.

Byron said...

Jollyreaper:
I don't see anything wrong with artificial gravity, so long as it doesn't become a reactionless drive.

The ease of landing on water depends on if you're just landing or if you're actually doing serious movement.

Lifting bodies are unlikely to be a winner. A vessel like this will have a very heavy wing loading, which means you'll add lots of drag, but not that much lift.

Milo:
If you don't mind being highly unaerodynamic, then a "flat" ship - with the engines laid out along the bottom side - would be easier to keep stable than a rocket shape. It would also give you a lot of space for engines.
The other problem is that it's structurally weaker then a normal vessel. However, there is an option. In the atmosphere, it flies sideways. In space, it flies forwards.

jollyreaper said...

@Byron

My suggestion regarding logistics is to decouple the combat skyships and the support vessels, unless the vessel in question operates independently. This would give a fleet ship a clear advantage over a patrol one, but that's to be expected.


What are you defining as the support vessel here?

I suppose the situation could be vaguely analagous the late Cold War USN. All the carriers were nuclear and we'd started building escort cruisers specializing in air defense that were also nuclear so they could have long legs like the carrier. The supply ships were all conventional, of course, but the expectation is they'd be meeting the fleet for resupply.

A strike fleet would require long legs and a great ability for independent operation. A defense fleet would be operating close to base and can concentrate on weight of firepower rather than long endurance. From what I've read, Soviet ships were supposed to be pretty miserable to serve on but had a ridiculous amount of firepower. USN ships are more comfortable, livable, and have good sea-keeping characteristics. Seemingly underarmed compared to the Soviet counterparts but more likely to actually be where they're needed and in fighting trim if the balloon goes up.

For maximum flexibility, the two fleets could share certain design types but the strike fleet would have an emphasis on ships that can be lived aboard for year-long deployments and would also have a support element that carries all the equipment to be a mobile shipyard.

The biggest difference between the fleet imagined here and the modern day is that guns are of more importance and guided weapons not as all-dominant. Imagine using kiloton shells as AAA against incoming missiles.

What this would mean is that size does still have some bearing and there's an advantage for a ship that can throw some shells and take some hits.

I really find modern warships to be boring. They all look like the crude geometric shapes from the early simulators. By contrast, WWII designs were a riot of details. The Japanese with their masses of gun tubs and pagoda masts were nuts.

http://www.aeronautic.dk/Warship%20Hiei.htm

Incidentally, here's a time waster potentially as destructive as TV Tropes: http://conceptships.blogspot.com

There's a really nice ornamental ship design here. Looks like something a decadent spacefaring aristocracy would come up with.

http://www.igorstshirts.com/blog/conceptships/2012/angel_a/angel_a_16.jpg

Anonymous said...

(SA Phil)

If the Q thruster actually worked - there would be massive research money being spent on it. We would be able to read paper after paper from well known universities exploring the applications.

Its like Cold Fusion and assorted technical oddities.

The modern version of Engineering Snake Oil.

Anonymous said...

=Milo=



Byron:

"The other problem is that it's structurally weaker then a normal vessel."

How so? We cracked the art of low, sprawling buildings long before we figured out how to make skyscrapers.


"However, there is an option. In the atmosphere, it flies sideways. In space, it flies forwards."

I did say the flying saucer wasn't intended for use in an atmosphere.

You are correct that if a flying saucer were to be used in an atmosphere, it would have to move horizontally like an airplane, acting as a lifting body (as real flying saucers did), to avoid suffering the atrocious drag that would come with trying to fly vertically with such a shape. (Vertical take-off is, in fact, one thing that rocket shapes are obviously best at.) This isn't how flying saucer UFOs are usually shown to operate in fiction, though.

However, I was specifically supporting the use of flying saucers for outer space, with the engines mounted perpendicular to the plane of the saucer, since drag isn't an issue in vacuum.

Byron said...

Milo:
How so? We cracked the art of low, sprawling buildings long before we figured out how to make skyscrapers.
Assuming a central engine. If you do that (one big engine in the middle) everything has to be cantilevered, which is bad. If you spread the engines out, it's a lot better. Probably should have clarified.

Anonymous said...

(SA Phil)

A lot also depends on how your anti-gravity works.

If you can vector your reverse gravity - worrying about thrusters becomes redundant.

Anonymous said...

Thinking about the Voidship concept, with its antigrav, instead of 'reversing' your antigrav device for internal gravity, why not just put a smaller version of the antigrav device in the celling of your hab section and let it push you down? I know you would need to brace the hull to a greater degree than otherwise, but now you don't need to fiddle with the way it works; keep it simple...

Ferrell

Rick said...

On antigravity: These a remarks I've made in previous comment threads, but I'm too lazy to hunt and link.

In principle, there's no reason why an antigrav ship should need to expend any power to *hover*. No work is being done, any more than if the ship landed on top of a tall building.

In practice, 'leakage' from the antigrav coils (or whatever) would probably cause the ship to gradually descend. And the released energy must appear somewhere, probably in the coils as waste heat that has to be got rid of.

To climb, feed power into the coils. The more power, the more rapid the ascent.


As the above implies, I picture an antigrav drive as having coils of some sort. Substitute evocative gizmo of your choice. (If you need McGuffinite, that reminds me of Cavorite, and has a very retro flavor. YMMV.)

Whether / how you can get horizontal motion from the antigrav gizmo is an interesting question. Possibly you cannot, and need thrusters / fans for horizontal motion.

When I tried to run some numbers, it turns out that getting into space even with antigrav drive would be ... challenging. You still have to pump energy into the field.

But if rifts are in the atmosphere, or even near space, reaching them could be a Whole Lot Easier.

(Are rifts a sort of subspace, not necessarily with FTL properties, but separate from ordinary space?)

Going *up* would

Rick said...

Good on Google for restoring email for comments!

Now, if they'd only provide a means for me to let Milo's posts through instead of having to bail them out of spam jail.

Anonymous said...

(SA Phil)

Actually an Anti-gravity device that completely ignores gravity - which you all seem to be describing..

Seems to me to be more impossible than an anti-gravity device that uses artificial gravity to counter gravity.

Since according to some Gravity is a property of space-time...

Ignoring space-time seems higher on the impossible scale than influencing it (which we have observed with black holes and such).

jollyreaper said...

I have no clue as to how antigrav could work. Frankly I'm confused by space-time being described as something that could curve but also communicates gravity effects with gravitons.

Dark energy is supposed to have an antigrav effect and pushes things apart. I'd consider that to be a candidate for the mechanism of antigrav but I don't have enough imaginary physics to do a proper job of explaining it.

As for my conception of the void.

What people have observed:
There are spots on the planet that serve as rifts. You can detect them from short range with the right sensors. Survey ships are employed to find them the first time. Any ship that activates the proper fields can pass through the rift. Without those fields you sail right past remaining in atmosphere.

The voids are not outer space. They are just that, voids. Physics is a little wonky. Electromagnetic radiation doesn't propagate the same as on a planet so you're stuck at short ranges with the eyeball. Even telescopic observation can turn to mush at a distance.

Void space is not connected to each other. You aren't entering THE VOID when you enter a void. The void between world A and B could be different from the one you encounter going from B to C.

There are common routes known of between nearby rifts within a void. Those routes would thus be the shipping lanes. The voidspace between worlds A and B might only have those two rifts that are used commercially. Survey ships moving really, really far out might find voids that connect to other worlds. Some might be seemingly useless but one useless world connecting to a useless space connecting to another could yield a path that nobody knew about. You in effect can find a backdoor to an enemy's important planet.

With these routes being a vital military secret, empires that trade might use stations in void space as a neutral ground for exchanging cargoes. One power moves goods to the station, the other power picks them up. The goods complete the trip but the route to the enemy's important world is never established.

Are these worlds existing in the same galaxy? In the same universe? It's hard to say. The evidence is not clear.

Real questions arise when the ruins of similar human civilizations are encountered. Where did these humans come from? It's fine if you're finding something that looks like nothing you've ever seen, properly alien. It's not so fine when you're finding what looks like ruins of a civilization similar to your own down to the styling choices on the ruins.

While there should never be definitive proof of this within the story, some scientists believe that the entire world they're living in is nothing more than simulation. And that's how you explain parallel societies. They're all randomly seeded from prior human history and playing out as alternative histories except histories that can intersect when new pathways are found through the void. What's the purpose of all this? Hard to say.

If it is indeed a simulation and routes through the void can change over time, then the encountering of a new civilization on a new route, one that is a good technological match which can lead to a protracted war, that looks less and less like coincidence.

Byron said...

I'm not going to speculate on the physics of antigravity. However, you can probably have any power level you want. Who knows what power losses you suffer.
(I'm not going to speculate, as I have to do that sort of thing far too often already.)

Brett said...

@Anonymous
Well, not so clear-cut. The "no stealth in space" thing applies across interplanetary distances, but trying to point a telescope at a solar system many lightyears out is more tricky.

I'm not even sure it applies at the interplanetary level - it really depends on the scenario. In reality, I suspect part of the difficulty would involve telling the infrared dots 50 million kilometers away that are enemy warships, from those that are friendly ships moving around a solar system. The setting where you can even have Space Opera-ish fleet actions is going to be one where humanity is all over the solar system (or a solar system).

I wonder if this would eventually lead to some type of Geneva-style convention, wherein warships have to broadcast a radio signal identifying themselves as warships of a certain polity, civilian craft, etc. Any non-broadcasting ship headed rapidly towards a population center would be assumed hostile.

Anonymous said...

(SA Phil)

If gravity curves space-time and is a direct result of mass as described by some -- then I don't think Gravitons are necessarily part of that scenario.

Essentially if a black hole can bend light, doesn't a tennis ball also bend light to a lesser degree? And if a black hole curves space time, doesn't it also follow that a tennis ball would also curve space time to a lesser extent?

I was browsing the Science/physics section recently and there is a book coming down pretty hard on the modern crop of theoretical physicists for going much too far with their hypotheses -- basically that they are straying down the path of non-observational conjecture --- not any different from religion.

I should have picked it up. It might have had some information on this topic and the possible existence or non-existence of gravitons.

Not necessarily applicable to your story setting of course since it describes a completely different universe.

Anonymous said...

Brett,

I'm not even sure it applies at the interplanetary level - it really depends on the scenario. In reality, I suspect part of the difficulty would involve telling the infrared dots 50 million kilometers away that are enemy warships, from those that are friendly ships moving around a solar system. The setting where you can even have Space Opera-ish fleet actions is going to be one where humanity is all over the solar system (or a solar system).

I wonder if this would eventually lead to some type of Geneva-style convention, wherein warships have to broadcast a radio signal identifying themselves as warships of a certain polity, civilian craft, etc. Any non-broadcasting ship headed rapidly towards a population center would be assumed hostile.
==============

Unless there are a truly massive number of ships - wouldn't you basically know where the friendlies are based on their mission profiles and flight plans?

With a Solar System you could have a pretty consistent communications net.

Kind of like the internet back in the Al Gore Days ... with transmission lag replacing tiny baud rates.

Although that could take us back to a variant on Rick's original scenario where a non-military "friendly" isn't necessarily quite a friendly as you thought ...



(SA Phil)

Eth said...

SA Phil said:

"If the Q thruster actually worked - there would be massive research money being spent on it. We would be able to read paper after paper from well known universities exploring the applications.

Its like Cold Fusion and assorted technical oddities.

The modern version of Engineering Snake Oil."

Yes, unfortunately.
But the concept can be interesting if you want quite hard SF settings, as you would have an interstellar-capable drive without the incredible engineering feats and massive fuel tanks for any unmanned interstellar probe. This would allow for more 'causal' interplanetary and even interstellar travels, without the need for a near god-like civilisation.
Oh, and those Q-thrusters have to run on cold fusion, obviously.


SA Phil said:

"I was browsing the Science/physics section recently and there is a book coming down pretty hard on the modern crop of theoretical physicists for going much too far with their hypotheses -- basically that they are straying down the path of non-observational conjecture --- not any different from religion."

We all know how many people treating religious texts as scientifically exact (instead of, say, allegories). Unfortunately, many also treat scientific theories as gospel truth in what to believe.
Going to the extreme, both can give us funny things, like creationism or quantum mysticism. Even funnier as one often mocks the other. Yet even funnier when it it for having 'irrational beliefs'.
But anyway, it's far from being a new trend, look at Pythagoras.

Also, why couldn't gravity be both caused by space-time deformations and carried by gravitons?


About the voids. How would they be 'shaped' like? Infinite? Closed on themselves, like hyperspheres? Surrounded by invisible walls? Surrounded by zones where laws of physics stop being stable, and your ship is disintegrated because strong nuclear interaction stops acting (integer overflow)?
If closed, what size would they be? And how many rifts one void could expect to have? Two? More? Sometimes only one (dead end)?
Also, if voids are meant to be big, how far would you have to be to detect a rift? If your sensors have only a range of 1km, it may be difficult to find a rift even only 1000km away...

Oh, and if there are pieces of rock up to the size of worlds in the voids, would there be settled ones? Void-born ecosystems? What would be their surface conditions?

Thucydides said...

The description of "voids" sounds suspiciously like that of a wormhole, without the superluminal speeds inside.

One of the best "Scratch Forces" books I can remember is Ben Bova's "Millennium". The US and Soviet Space forces are engaged in setting up ABM shields for their respective nations, but also space jocks poke among the decoys and blow up opposing ABM satellites. (A grenade launcher shattering the optics is usually sufficient against an inactive satellite).

The scratch part of the force comes when the Russian and American "Theater" commanders on the Moon realize that:

a. This is incredibly dangerous for themselves and the troops under their command, and;
b. Whoever wins is going to blow up the Earth.

By joining forces they hijack and integrate the two separate ABM networks and impose a sort of peace on the Earth, where they can reduce or eliminate ICBM threats.

Now the scratch comes from jury rigging their separate units and networks into one, and also in becoming the "third party" in the conflict, as opposed to being parts of the rival Great Powers. In terms of hardware and software, the only way to get military forces into space is for the military to decide to go into space; commsats and other non milspec space hardware in this scenario would be vaporized quite easily by the ABM sats once activated.

I think the book cuts to the core of the matter; Space is such a hostile environment that the people actually living and working there would have little interest in adding to the danger. Any space forces in the PMF or otherwise will be created and maintained by the Imperial metropoles, not by the colonies themselves.

jollyreaper said...

@brett

I wonder if this would eventually lead to some type of Geneva-style convention, wherein warships have to broadcast a radio signal identifying themselves as warships of a certain polity, civilian craft, etc. Any non-broadcasting ship headed rapidly towards a population center would be assumed hostile.

This makes a lot of sense. You have to learn the rules of the road for the systems you visit.

Offhand, I'd wager there's a max delta-V and max velocity in certain zones, forbidden trajectories, and transponder requirements. Certain behaviors will get you intercepted or possibly destroyed. You want to stay in one piece, you play by the rules.

jollyreaper said...

@sa phil


I was browsing the Science/physics section recently and there is a book coming down pretty hard on the modern crop of theoretical physicists for going much too far with their hypotheses -- basically that they are straying down the path of non-observational conjecture --- not any different from religion.


Please let us know if you remember the name. Sounds fascinating.

Byron said...

Brett:
I'm not even sure it applies at the interplanetary level - it really depends on the scenario. In reality, I suspect part of the difficulty would involve telling the infrared dots 50 million kilometers away that are enemy warships, from those that are friendly ships moving around a solar system. The setting where you can even have Space Opera-ish fleet actions is going to be one where humanity is all over the solar system (or a solar system).

I wonder if this would eventually lead to some type of Geneva-style convention, wherein warships have to broadcast a radio signal identifying themselves as warships of a certain polity, civilian craft, etc. Any non-broadcasting ship headed rapidly towards a population center would be assumed hostile.

Not really. It's just data processing. After all, the situation isn't "boot up the system and take a look around". It's continuous scan, so every one of those dots will be tagged as soon as it's detected, and tracked from then on. And the characteristics of the contact would reveal quite a bit as well. I wouldn't be surprised if there were standard transponders, but it's not like not having it would result in mass chaos.

Thucydides:
The scenario you describe sounds less like PMF and more like PNF (plausible near future). If we have significant interplanetary trade and colonies large enough to have wars, they by definition are more settled and in a less precarious position. Which can and will lead to some form of conflict.

jollyreaper said...

@Eth


About the voids. How would they be 'shaped' like? Infinite? Closed on themselves, like hyperspheres? Surrounded by invisible walls?


I think the best way to describe them would be contradictory.

There's an online game called EVE that features lots of ship combat. You have full solar systems simulated with combat zones you can enter when running missions. Terminology from conventional dungeons and dragons games is to call these things dungeons and the parts of each dungeon rooms. But it sounds completely contradictory to think of an empty space with no walls a dungeon or to be comprised of rooms. Some of these dungeons are instanced so nobody else but allies could enter them with you and others remain in the world so human enemies could crash your party for some player-vs-player pew-pew.

The voids will appear infinite but at the same time they're distinct from each other. How do you have stacked infinities? Where are the worlds you're traveling to? It's the next system over through a void but where is it? Can you detect radio transmissions from your own home planet in the sky of that one? Where are you? Is it the same universe, a parallel universe?

It remains maddeningly imprecise to nail down.


Surrounded by zones where laws of physics stop being stable, and your ship is disintegrated because strong nuclear interaction stops acting (integer overflow)?

I love that idea and used it to describe hyperspace in a different setting. You have to have a hyperfield around your ship to keep from being destroyed. Any probe you push beyond the field disintegrates because of exactly that sort of thing. Therefore it's impossible to know anything about hyperspace except what you can read from ripples and currents bumping against your hyperfield. You can't stick a camera out and take a look.

If closed, what size would they be? And how many rifts one void could expect to have? Two? More? Sometimes only one (dead end)?

They appear infinite. Nobody has ever found an end, looped around to the beginning, etc.

I think that the diffraction of light through the different spaces would provide colors and character to the void spaces. Artistically, this would give them a mood. Sort of like the colorful space of Babylon 5, moody starscapes in the Homeworld game.

The rule of thumb is that there are always more rifts but the question is how far are you willing to go to find them?

The difficulty is you have to do a three-dimensional expanding box search because you have to get close to a rift to detect it. I'm thinking that a mothership would deploy remote sensor clusters and thus sweep vast areas for rifts. Or maybe the ship just spins gently and lets detectors spool out on long cables. A ship doing a run has long strings of detectors spooled out and sniffing for rifts.

jollyreaper said...

Also, if voids are meant to be big, how far would you have to be to detect a rift? If your sensors have only a range of 1km, it may be difficult to find a rift even only 1000km away...


Not entirely sure. One of the goals is to keep the distances at a useful, cinematic range but also have really long runs when the demands of drama need a long transit. Commercially useful rift courses minimize transit times and strategically useful rift courses can take you places your enemy didn't think you could go.

Oh, and if there are pieces of rock up to the size of worlds in the voids, would there be settled ones? Void-born ecosystems? What would be their surface conditions?

The thinking I had is that you could have the ruins of worlds in there. Here's a hundred square miles of cityscape floating through the void. The assumption is that it was pulled through a void from a proper planet in some sort of accident.

But at some point in the story there would be an "oh shit" moment when a planet becomes unstable and the characters have to escape. The whole planet is coming apart, the sky is changing colors and the ships are lifting up and clawing for altitude when the ground falls away and below is nothing but void.

So, what did they just see? Did they just see a planet collapse into a void? A solar system? An entire universe? Rifts always have a planet-scale body on one side and a void on the other. What they just saw implies new rifts have opened up and old ones have closed. How do they find a rift to get back to a real planet?

There's an appreciation that rifts can change in time but this is quite rapid. So does this mean that a ship heading towards a known rift through a void might find nothing on the other end? There's not a huge fuel reserve fraction. If you run out of fuel in the void, you die.

One other thought. Some rifts could appear in the upper atmosphere of gas giants. That would be one hell of an environment to be fighting in.

Anonymous said...

=Milo=



Byron:

"Assuming a central engine. If you do that (one big engine in the middle) everything has to be cantilevered, which is bad. If you spread the engines out, it's a lot better. Probably should have clarified."

Ah. I was intending to have several enginges - one of the advantages of a large floorspace is that you have room to do that.

Of course, several small engines might be less mass-efficient than one large one, which poses different problems.



Brett:

"In reality, I suspect part of the difficulty would involve telling the infrared dots 50 million kilometers away that are enemy warships, from those that are friendly ships moving around a solar system."

If you can't keep track of your own ships, you're doing something wrong.



SA Phil:

"Although that could take us back to a variant on Rick's original scenario where a non-military "friendly" isn't necessarily quite a friendly as you thought..."

This is one scenario where the no stealth in space thing falls flat!



Eth:

"We all know how many people treating religious texts as scientifically exact (instead of, say, allegories)."

Which, I'll note, is how most of those religious texts were originally meant to be read.

I don't much care for people (or groups) retconning "Oops, I guess that was an allegory! But I still believe in it." AFTER their beliefs have been shown to be wrong.

Byron said...

Milo:
Ah. I was intending to have several enginges - one of the advantages of a large floorspace is that you have room to do that.

Of course, several small engines might be less mass-efficient than one large one, which poses different problems.

Yes. However, economies of scale, not to mention things like plumbing, conspire against that, too. Also, it leaves (presumably) a bigger target for the enemy. No matter how you do it, the engineers will be rolling their eyes and/or calling their investment advisers.
It's not impossible, but it strikes me as a bad idea for most uses. Except the aforementioned airplane/spaceship, which is of dubious utility.

Anonymous said...

Brett,

Also, why couldn't gravity be both caused by space-time deformations and carried by gravitons?
===========

I suppose it could. I am not a Physicist so my understand is definitely second hand from reading.

One model Gravitons do work in is String Theory which does seem to be the FOTM.

The main reason for guessing that Gravitons exist is all the other "forces" of the Universe have elementary particles associated with them.

Anyway I am not saying anything is necessarily how things work just that Gravitons aren't necessarily required for Gravity ... especially since we have not observed them to actually exist.

But really I think that Gravity is a result of Mass. Its the how/why that is the tricky part.

(SA Phil)

Anonymous said...

Ahh that was Eth* not Brett- sorry

(SA Phil)

Thucydides said...

The rifts scenario deserves a thread of its own (or maybe your own blog so we can invade); we are dealing with all kinds of alternative universe/physics questions that could take hundreds of threads to work out.

WRT PNF and PMF space forces, we really need to find much more efficient ways to get around, especially ways to reduce the amount of mass needed to propel the ship and protect the crew. Here are two ideas culled from NBF:

http://nextbigfuture.com/2012/03/potential-for-ambient-plasma-wave.html

http://nextbigfuture.com/2012/04/nasa-active-radiation-shielding.html

Of course, you still need a honking reactor to power all this stuff as well as the weapons and sensors, but smaller, lighter spacecraft will lead to more spacecraft, more economic activity and more potential reasons for conflict.

Given the dangers of living and working in space, the colonies may work on means of conflict suppression in preference to actual fighting machines. If ambient plasma wave propulsion is feasible, perhaps some means of temporarily negating the plasma fields near the planet might be appropriate ("Beg to report sir; the drive's not working, but an enemy rocket powered kinetic energy weapon is on an approach vector. Their constabulary will divert it if we strike the colours now...)

Anonymous said...

(SA Phil)

So going back to the original Post...

Makeshift combat might actually be fairly common.

Two scenarios I can think of.

The First would be where neither side has any purpose built warships.

The Second would be where one side has a dominant military and the other is fighting a guerrilla type war. This is not that much different than Iraq with IEDs.

As to weapons - the Science survey vessel might be equiped with Probes for launching towards items of interest. A crafty engineer on board might be able to repurpose those into kinetic kill missiles.

They already have guidance, propulsion, etc.

Another weapon that presumably any hydrogen powered group could make would be a light gas gun / or Combustion light gas gun. This would be a very effective hyper velocity weapon at the ranges you might see in orbital/near orbital conflicts.

A fuel Tank could potentially be re-purposed with the right added plumbing. These weapons can exceed 10 m/sec.

Another even more makeshift weapon is something I used in a short story I wrote about 20 years ago where a former space battleship/museum ship was attacked by revolutionaries. The Shuttles used to transport tourists were reprogrammed by a crafty student to use as short ranged missiles. The element of surprise was the main advantage to those.

jollyreaper said...

@Thucydides

Yeah, might not be a bad idea to have a blog for that sort of thing. Work out the kinks on different ideas.

One thing I really like that Winchell does is take good ideas and cull them from the thread chaff and put them in a nice, indexed fashion along with the appropriate topics.

http://jollyreaper-ideapit.blogspot.com

I'll throw my off-topic and thread-drifty thoughts over there. :)

Thucydides said...

Looking forward to following up on your blog!

(Don't neglect Rocketpunk, though)

jollyreaper said...

Of course not. :) It's easier to keep up with again since they put back the email comments.

The way my brain works is really awful, though. Took a nice long ride yesterday with the intention of trying to think through a few stories I'm stuck on. The result? Brand new outline for something else entirely unrelated.

Honestly, I think this is what happens with fantasy authors. Rather than starting up a whole new novel in a new setting, they're starting up a new novel in the same setting and in the same time but with completely different characters. So a story that could be told in one damn book if they stuck with one damn set of characters becomes a sprawling mess that they won't manage to finish writing in a single lifetime.

I can sympathize.

Eth said...

Milo:

"Which, I'll note, is how most of those religious texts were originally meant to be read.

I don't much care for people (or groups) retconning "Oops, I guess that was an allegory! But I still believe in it." AFTER their beliefs have been shown to be wrong."

Short answer : Not really, no.

Long answer : ...may not be the best place for it.


One example coming to mind is the game Shattered Horizon. Now, it's not the hardest SF out there, and it's arguably more PNF than PMF, but some points are interesting.

Basically, you have the ISA (International Space Agency, the space police) and the MMC (Moon Mining Consortium, mostly for lunar He3), the former watching over the latter.
Until a good chunk of the Moon blows up and creates an asteroid arc around the Earth.
(The devs said that they had done the calculations about that, so they probably had something in their mind other than spontaneous He3 fusion, but we'll probably never know what.)
Both forces are more or less cut from the Earth, the MMC survivors trying to continue survive and the ISA trying to arrest the MMC survivors. Which turns fast into a full blown war.

They only have rifles, used as police weapons (and later improvised rifles), so it's space infantry warfare. Also, the explosion and subsequent debris damaged most of their infrastructure, sensors included, so guerilla warfare becomes prominent, as each side has only limited sensors and movement, in a field of debris where they try to secure various vital objectives.

One map specifically features a giant magnetic catapulte, previously used to send cargo to Earth from lunar orbit, precisely because it could be used as a formidable weapon.

Now, some concessions to realism are made, to keep it playable. For example, why aren't ships used to send kinetic attacks against some targets, why micrometeoits kill you only when you get away enough from the piece of rock you're fighting for...
But those could probably be changed or handwaved in other media.

The interesting part is, they are stranded away from any help, with damaged equipment and sensors, and have to fight with light police weapons instead of far more effective military ones.
The situation would probably resolve or at least evolve quickly (like, say, once one side secures the catapults), but it gives for a short time things as strange (and potentially interesting) as infantry space warfare.

Geoffrey S H said...

If you are beseiging a large airless planet/moon with extensive fortifications and bunkers, then perhaps something like infantry space warfare might be called for...

I.e.: You can't starve them out (due to internal hydroponics bays) and bombardment will only get you so far.

Thucydides said...

Unlike Earth warfare, there are few limits to the amount of energy you can deploy against bunkered targets.

We have seen news reports of a super bunker buster bomb being deployed to US forces in order to threaten deep facilities like Fordo, but only a B-2 or B-52 can carry one. Any deeper or harder bunkers can only be attacked by nuclear weapons since there are no aircraft capable of carrying larger bunker busters.

WRT deep bunkers on the Moon, you can accelerate KE projectiles to pretty incredible speeds due to lack of air resistence, and go as massive as you want (limited only by your energy budget).

The giant laserstars that appear in many of these threads can steadily drill down to deep bunkers, given the willingness of commanders to spend time.

Or you can nuke them from orbit. It's the only way to be sure...

Anonymous said...

(SA Phil)

You are still limited in destructive energy.

A few limits I can think of
-Your heavy lift capacity (for the nukes or possibly propellant)
- You propulssive capacity (for launching rocks from SPACE)
-The bunker's definsive capacity (Anti-kinetic defense of whatever type)

Example -
If you have a 10000 kg rock but only a small amount of hydrogen .. the war might be over before your efficient solar electric drive acclerates it to bunker busting status.

Thucydides said...

The ORION anti asteroid weapon mooted here several times can hurl a 3 ton structure (after all the nukes have been fired) hard enough to cover 15 million Km in five hours and strike with a Gigaton of energy, which will ring anyone's bells. As a "scratch force" weapon, the Earth might hold a few as part of an asteroid deflection squadron, but decide that some space activity is an existential threat to either the owners or the Earth as a whole. Something like this could essentially cover most of the Solar system (although the lag time might be excessive if the threat is in the outer solar system). Reprogramming to save a few "physics packages" for terminal manouevres would make this a pretty terrifying weapon.

The ultimate example of a KE attack in literature is probably the final battle scene in the "Forever War", where a fighter slingshots around a collapsar and sends two missiles at relavistic velocity against the alien spaceship. One hits, and turns the ship into plasma, while the other one glides past and strikes the planet, causing a massive earthquake and collapsing the base.

Anonymous said...

Thinking about surface warfare, I believe it would be more like light mechanized infantry; foot soldiers riding to battle in lightly armed vehicles. Probably modified construction or exploration vehicles with improvised weapons. I would expect that most fighting would be close to colony sites, or strategic resources; Somehow, the thought of two colonies lobing artillery shells at each other is a recipie for disaster.

Ferrell

Thucydides said...

Might not even have to be modified greatly. The IDF has found bulldozers to be quite useful when engaging in urban combat; besides armouring the cab and applying "birdcage" screens to detonate RPG's they are pretty standard machines.

The Infantry soldiers follow closely behind as the machines demolish bunkers, roadblocks or breach walls, they are right at hand to exploit opportunities and also there to protect the bulldozer from rude people with weapons.

Tanks and heavy APC's provide integral fire support to shoot in attacks or shoot back, but these machines are the handmark of a modern mechanized force, not a scratch force.

Anonymous said...

If surface vehicles are already used for long-range exploration, they might carry collapsible pressurizable 'tents' for when expeditions set up camp, since it might not be comfortable to eat and sleep in the vehicles. These could be used for ad hoc military missions as well.

R.C.

Anonymous said...

=Milo=



Tents are fragile. If they're the only thing standing between you and vacuumy death, that's a problem.

I can believe that tents might be made which are reliable enough and airtight enough to hold up under peaceful conditions, but not while being shot at.

Byron said...

Milo:
Tents are fragile. If they're the only thing standing between you and vacuumy death, that's a problem.

I can believe that tents might be made which are reliable enough and airtight enough to hold up under peaceful conditions, but not while being shot at.

I'd imagine that the 'tent' is more like a transhab then a conventional tent. And those seem like they'd work well enough.

I'm not sure how improvised ground combat would work. Guns are easy to make with the sorts of materials found on any reasonable-sized colony, but ammo might be a different story. The difficulty varies based on how developed the colony is and how quiet the armament program needs to be kept. The biggest problem is a shortage of organics. Charcoal, for example, is unlikely to be a household item. At the same time, fertilizer manufacture sometimes involves sulfuric and nitric acid, and glyerol should be readily available, giving the components for nitroglycerine.
The point is that ammo might be a bigger problem then the weapons themselves, particularly given that recovering the volatiles involved would be quite difficult, and I doubt that fumes would be welcome inside the sealed environment.

Thucydides said...

For a scratch force fighting against a similarly equipped enemy, having a limited number of small arms (or support weapons for that matter) might not be such a big deal. You are not going to be outgunned unless you make a serious tactical or strategic mistake, nor will you be able to generate overwhelming force unless they make a mistake.

If you really want lots of relatively effective weapons, start cranking out strips of spring grade steel and manufacture crossbows and catapults. The crossbows should defeat spacesuits (14th century crossbows could take down a knight in full plate armour) and even light shelters or vehicles (especially if you target a vulnerable spot), and catapults can be made to defeat anything bigger and heavier (but not mmilspec).

Of course, other improvised weapons can be imagined; gas guns, light rail guns and coil guns might be possible (especially on vehicle mounts) and such weapons mounted on a shuttle or OTV would turn it into a scratch fighting vehicle for the "Navy", or to provide fire support to ground troops.

I'm still not convinced that people who live in insanely hostile environments would want to make life even more dangerous. If I were to write a scenario, the various colonists on Europa are banding together to build these things against the meddling bureaucrats and their police force from Earth...

Stevo Darkly said...

I'm sorry I didn't respond to this earlier. Since Milo's comments keep disappearing, I guess I don't see a lot of them, unfortunately.

Regarding the question of whether survey ships would be armed, this seems worthy of a riposte:

Stevo: "Ferrel did say "interstellar", and so your "no one" implicitly means "no one from our own civilization". There remains the possibility of hostile natives, or an aggressively expanding alien empire."

Milo: "Unlike Earth, though, we aren't expecting every nook and cranny of space to already be settled by natives."

Maybe not ... but the odds of such may be higher for those particularly Earthlike nooks and crannies that human explorers would be most attracted to. (And there might be threatening life-forms that aren't necessarily intelligent, technological natives.)

Besides, there's quite a gap between "danger in every nook and cranny" and a non-negligible possibility of danger. It's not like every nook and cranny in Africa has a lion crammed in it -- but if I were setting off across an unexplored savannah, I'd still really like to be able to carry a weapon with me.

Milo: "Unless we actually know there to be hostile factions operating in that region of space, "just-in-case" defense doesn't seem like a high priority."

I'm afraid I have to disagree. If I were the guy in charge of exploring any region of space where life might be found, I would require some "just in case" weaponry. I don't mean that every ship would have to be an armed-to-the-teeth super-duper-dreadnought-battlewagon ... but going into the unknown completely defenseless seems mighty unwise.

Addendum: It appears that Roald Amundsen took weapons with him during his expedition to the South Pole -- at least he writes that the expedition's ship took on a load of "no less than half a ton of gun-cotton and rifle ammunition" on the way to Antarctica. As it turns out, the interior of Antarctica is devoid of any threatening creatures bigger than a bacterium -- but Amundsen didn't know that, and I don't blame him a bit if he decided to lug some rifles along.

Anonymous said...

=Milo=



I'd say that personal weapons for use on the ground are more useful for an exploration party (and also cheaper and easier to carry) than spaceship weapons.

Getting attacked by alien lions is a concern on any lifebearing planet. Getting attacked by alien lions with spaceships isn't going to happen much.

Byron said...

Thucydides:
The thought of crossbows did cross my mind earlier, but I was too busy researching the components of gunpowder.
That said, yes, it is entirely possible. Nobody's ever built, to my knowledge, a modern military crossbow. I might go with cannons for heavier weapons, but the point is a good one. My understanding of the benefits of early firearms over crossbows was ease of learning, but changes when the amount of gun ammo is sharply limited.

Stevo:
I seriously doubt you could blunder into a spacefaring enemy over a short hop. And over a longer hop, what good is being armed? Unless you have a lot of weapons, you're almost certainly outgunned, and all those weapons cut survey capability and add cost.

Byron said...

A farther thought on crossbows:
Any crossbow for use in vacuum would have to be spin-stabilized, which, while not impossible, is not something that strikes me as easy, nor are there lots of plans floating around. Not a huge problem, but it would have to be dealt with.

Anonymous said...

=Milo=



Byron:

http://en.wikipedia.org/wiki/Crossbow#Modern_military_and_paramilitary_use

They've been used in a few places, particularly special forces, due to being quiter than guns.

The article also claims the Chinese security uses them "to stop persons carrying explosives without risk of causing detonation".


"Any crossbow for use in vacuum would have to be spin-stabilized,"

Do you need stabilization at all without an atmosphere? There's no aerodynamic effects to keep your bullet/bolt from travelling in a straight line, so conservation of momentum should apply - an object in motion stays in motion in the same direction.

Anonymous said...

No, the tents probably wouldn't be bulletproof. This is a scratch force using civilian equipment, after all. However, there are ways of reducing their vulnerability. Don't make camp too close to the enemy base. Laager the vehicles around the tents to give an extra layer of protection. You could also build a non-airtight protective 'igloo' around the tents, using metal plates or perhaps sandbags filled with regolith.

R.C.

Eth said...

About the crossbows, I'm pretty sure several special forces across the world use it as a silent weapon.
But with more advanced materials, are there ways to build tension-propelled weapons more effective than crossbows?

Anyway, the simple image of believable space crossbowmen (so wookies not included) is quite awesome, there may be interesting things to do with it.

Byron said...

Milo:
I misspoke. They do use them, but guns are obviously much better for most uses.
Actually, I'd expect these to be more primitive then modern composite models. That's not the sort of thing you have laying around.

Do you need stabilization at all without an atmosphere? There's no aerodynamic effects to keep your bullet/bolt from travelling in a straight line, so conservation of momentum should apply - an object in motion stays in motion in the same direction.
You still want to hit pointy end first. The problem is more one of the direction it's pointed in instead of the direction of motion. Launch will like as not set up some sort of spin.
The problem with that is that the orientation will remain fixed, instead of always being pointy end first. So if you shoot it straight up, it'll come down backwards. This might make it a good idea to use a spherical projectile. You don't care about orientation, and there's no aerodynamic effects to make it less desirable. Penetration might not be so good, but that's less of a problem.

Eth:


No, the tents probably wouldn't be bulletproof. This is a scratch force using civilian equipment, after all. However, there are ways of reducing their vulnerability. Don't make camp too close to the enemy base. Laager the vehicles around the tents to give an extra layer of protection. You could also build a non-airtight protective 'igloo' around the tents, using metal plates or perhaps sandbags filled with regolith.


I expect these tents to resemble <a href="http://en.wikipedia.org/wiki/TransHab>TransHab</a>, as I wouldn't want to go out in anything as flimsy as a modern camping tent. They're resistant to micrometeorites, which means that they're likely bulletproof, or at least bullet resistant, as well. And even if a bullet does get in, I'm positive that there won't be a catastrophic rupture. Slap some duct tape on it, and fix it later.

Anonymous said...

=Milo=



R.C.:

"Laager the vehicles around the tents to give an extra layer of protection."

...Hey, that's actually a good idea.

Did we just invent Circling The Wagons ON MARS!?


"You could also build a non-airtight protective 'igloo' around the tents, using metal plates or perhaps sandbags filled with regolith."

Yeah, okay, you're right. Using local materials to build fortifications when setting camp would probably work pretty well, and has often been done by real militaries.

I retract my objection.

Thucydides said...

This discussion is bringing out the inner condottiero in me...

Crossbows made of composite materials would seem to violate the "scratch force" spirit of the discussion; our heros need to get armed quickly with whatever is available. Steel springs might be part of the various vehicle repair kits, or making spring steel might be much quicker and easier than brewing up some sort of composite material (especially the really useful ones like Kevlar, Spectra or various forms of carbon fiber, along with the binding resin or whatever is being used to keep it together).

Torsion catapults are indeed more powerful than tension bows, but are also quite a bit harder to make and use (the torsion bundles would need special attention to ensure they are in good shape, have even amounts of tension etc.) Even more powerful would be counterpoise engines (Trebuchets), but as anyone who watches the pumpkin chucking competition knows, "tuning" a trebuchet for maximum power and smooth release is also a very skilled operation.

Spherical projectiles are a possibility, as well as various "darts" shaped like 12 gauge deer slugs, with much of the mass packed in the nose so they are roughly teardrop shaped.

Finally, crossbows are relatively simple to learn, which is why they took over the European battlefields so quickly. Welsh longbow men and Turkish Janissaries needed a lifetime of training to fire their longbows and composite recurved bows and create devastating arrow storms, while crossbowmen could be drafted and trained with minimal time and still take down an armoured knight. Firearms were equally simple to learn but much more powerful (they could hit at a longer range and no practical amount of armour could stop them). Japan could use social structure and geographic isolation to ban guns, and the Ottoman empire had a very large population of cheap manpower to expend on long term archery training, but in most places, the economic advantages of using large amounts of untrained manpower in a militarily effective manner (scratch forces if you will) outweighed all other considerations.

Anonymous said...

=Milo=



If you need something really easy to manufacture, how about slings?

They've been used by civilian rioters, and some more organized militaries have used slings to lob grenades and Molotov cocktails rather than rocks.

You're going to want to avoid using blowdarts while wearing a spacesuit, though :)

Geoffrey S H said...

Byron:

"My understanding of the benefits of early firearms over crossbows was ease of learning, but changes when the amount of gun ammo is sharply limited."

Also remember that a long-bow/ crossbow, while with modern materials might be less fragile than their predecessors, and possible just as powerful as some sidearms, would nevertheless be higher/wider than firearms, which could be crippling in many situations. The fact that cable/sinews/whatever are present (i.e.: you have a weapon with "string" attatched rather than a squat cylinder/cube shaped device)might make it easier to damage if handled roughly.

That said, history, in the very long term, can be cyclical about these sorts of things... ;)

übertronic said...

I would point out that in the nature of in deep space weaponry, laser weapons would be preferable for a variety of reasons. One would be the theoretically unlimited ammo, and another that you don't need to waste valuable mass and space carrying hunks of metal to lob at enemy stations or vessels. Orbital combat would be much different, due to the lack of need for large amounts of delta V. This would permit the usage of the much more efficient hypervelocity weapons, as ammo capabilities would be much less of a problem.

On a related note, if by means of [magitech] or [technobabble] you could produce an electrogravity well, capable of inducing artificial gravity field on the same scale as a large moon, you could theoretically fire one into a deep space warzone, and then fire your orbital combat vessels into orbit with an enemy ship. This would allow for usage of more practical weaponry without as much waste heat, while not consuming delta V.

Byron said...

Ubertronic:
I would point out that in the nature of in deep space weaponry, laser weapons would be preferable for a variety of reasons. One would be the theoretically unlimited ammo, and another that you don't need to waste valuable mass and space carrying hunks of metal to lob at enemy stations or vessels. Orbital combat would be much different, due to the lack of need for large amounts of delta V. This would permit the usage of the much more efficient hypervelocity weapons, as ammo capabilities would be much less of a problem.
The balance between weaponry is not as cut-and-dried as you think. For example, if we have only chemfuel rockets, but very powerful lasers, then lasers will dominate everywhere. Theoretically, your statement is true, but it's also a simplistic view of the situation.

Anonymous said...

If your colony doesn't have the industrial wherewithall to build a super-laser, then you won't have any; you can only use what you have on hand; adapting your existing technology to weapons is probably your best bet (if you need weapons, generally you need them now!), instead of taking several months/years to slowly go theough several generations of inproving your technology and industial base. Rockets, improvised projectile weapons, etc. will probably be the first generation weapons fielded to equip a 'scratch force' type military.

Ferrell

Anonymous said...

(SA Phil)

Gunpowder wont be necessary- you can use compressed air (or CO2, nitrogen or oxygen, etc) to make a weapon with comprable performance characteristics to medium power firearms.

So you would just need a machine shop and a gas compressor. Gas handling would be a required tech component of any colony.

Take a look on You Tube there are air rifles which pierce cinderblock, used for hunting game, etc.

Byron said...

Phil:
That's a really good idea. I know that at one point, air rifles were used as military weapons for snipers. And oxygen is easy enough to get from most rocks. Plus, it avoids gumming up the air when fired inside.
The biggest problem is that air guns are limited to about the speed of sound, which means pistol velocities. According to Wikipedia, the big problem is that while they can break the sound barrier, the pellet goes transonic very quickly. That's not a problem in a vacuum.
So weapons would go from crossbows to air rifles. Interesting.

Anonymous said...

(SA Phil)

For deep space I think the Laser's effective range is the big determiner in all this.

If your engagements are are close range ALA Battlestar Galactica you dont need a laser you could just throw rocks.

At short ranges like <1km or less a laser isnt really better than a projectile weapon since it produces so much heat compared to total transfered energy.

At long ranges- lasers rule - because lightspeed is a bit hard to compete with in a setting using reaction drives with realistic propulsion. If your missle takes minutes to cross through a laser's effective kill range, it is not going to work unless you have a whole lot of missiles.

Byron said...

Again, the weapons balance is more nuanced then statements like "lasers will dominate at long range" imply, because there are so many tech assumptions involved. First off, how accurate are the lasers? Can they hit a target a light-second away? If so, can they do any damage? What's the specific power? Efficiency? Cycle time?
What about kinetics? Do we have something better then modern chemfuel for missiles? How easy is it to detect a projectile? How quickly can spacecraft dodge? What's the efficiency, specific power, and so on of coil/railguns? How much do they cost compared to each other, and how will it be used?
All of these, or at least a lot of them, need to be considered. And I didn't even mention drives, economics, and so on.
And Phil, at less then 1 km, you're touching. Point-blank range is more like 100-500 km, minimum.

Anonymous said...

(SA Phil)

Byron, all true -- I meant basically it depnds on the effective range of your laser weapon.

If it is only 1 km ... forget it, I may as well use a machine gun.

The close range was an homage to current opera where the fights take place at thrown rock ranges.

there is also a typo - I had meant <1000 KM not <1 km.

At some range ... depending on a lot of things like you mentioned ... the laser's Lightspeed advantage becomes really hard to beat.

Thucydides said...

While in theory you could gang many small lasers to create a large beam emitter (apparently the plan of Lasermotive http://lasermotive.com/, but you still need a high quality mirror and a very accurate pointing system to aim the beam.

A solid base or a massive spaceship structure to keep the optical train aligned, and a high quality sensor suite is also a must if you plan to attack targets at long (light second) ranges.

A scratch force might be able to improvise something if they have an existing laser launch system (a la Lyek Myrabo); orbiting "fighting mirrors) provides a wider arc of coverage for the beam, but probably only enough to clear targets out to GEO.

Thucydides said...

A sort of twist here; using this design (which our plucky rebel alliance forces could build from the parts bin) we could get a theoretical ISP of 250,000 seconds. Only the Imperial warships from Earth with their fusion drives could match that, and they are far more rare and expensive...

http://www.lpi.usra.edu/meetings/nets2012/pdf/3045.pdf

If a scratch force is to have any sort of credibility it needs to be able to face the enemy on somewhat equal terms, especially in a "Naval" setting. Naval asymmetric warfare is a result of special circumstances. Don't forget what ended the reign of pirates everywhere was the willingness of real navies with real warships to go after them in force and root them out both at sea and chasing them down to their bases (The Roman navy was perhaps the first large scale example, but John Morgan did the same as Governor of Jamaica and the US Navy hunted down the Barbary pirates "on the shores of Tripoli".

Anonymous said...

Just read the article about the "HIIPER" plasma engine; it looks promiseing, if it can live up to its potenial. It seems like a "Sub-torch" type drive; we might have some aspects of a Rocketpunk future after all...

Ferrell

Anonymous said...

(SA Phil)

With a suitably energetic drive ... your transport ships could be effectively transformed into drones with short ranged direct particle weapons.

Just accelerate them in and then turn them around and decelerate to fire.

jollyreaper said...

A new website not to be named!

http://www.scifiideas.com/

looks ADDICTIVE.

jollyreaper said...

Evil Industrialized Aztec Death Cult from the Void and Rifts setting. Wrote up some ideas in more detail.

http://jollyreaper-ideapit.blogspot.com/2012/04/warrior-race.html

Tony said...

Asking forebearance for my earlier blowup -- for which: mea culpa, mea culpa, mea maxima culpa, specifically to Rick, jollyreaper, and anybody else who I can't think of but offened...totally my fault for thinking myself bigger and smarter than others.

--------------------

The only thing that I'd say is that we shoudn't be so sure that warfare is about destroying things. If forces are for any length of time field expedient, they will likely be either grossly lethal (ramming a torchship into a space station or asteroid) or not very lethal at all (throwing rocks and odd junk at each other, without much, if any, guidance, and with little relative velocity). The former may not be any more cost effective than the latter, given the expenditure of capital equipment.

Also, we shouldn't think of the objectives as being something that we necessarily want to destroy. It wouldn't surprise me that early forays into space warfare involve essentially police or security -- or even amateur filibusters -- taking over objective installations "on the ground" as it were, with a minimal use of outright violence.

Then, of course, one sees purpose built interceptor spacecraft to keep enemy raiders away. These are countered by counter-interceptors. The interceptors get bigger, more maneuverable, large in number. So do the counter-interceptors...

Byron said...

Tony:
Welcome back.
And for once I can say that I totally agree with you.
I actually don't have much to add, except that the more I think about it, the less likely lasers seem as an improvised weapons system. There is simply no place for laser weapons to come from. Industrial uses lack the optical train, and scientific uses are unlikely to be hardened enough for weapons use. The only caveat is if either some sort of laser beam propulsion, or some kind of antimeteor system is in use. Both of those are likely to be fixed, which makes attacking the enemy directly even less likely.

Anonymous said...

=Milo=



Byron:

"Both of those are likely to be fixed, which makes attacking the enemy directly even less likely."

Don't forget the value of surface-to-orbit weapons. Being stuck on a planet isn't that huge of a disadvantage as long as your gun is big enough.

Byron said...

Milo:
That was my point, though I could have phrased it better. If the enemy's base is impervious to any attack you can mount, you won't attack it. That could make for interesting story constraints.

Rick said...

Welcome to a new commenter! And a 'returning' one.

I don't have a Whole Lot to add here. But Tony's point about warfare not being just about blowing stuff up is crucial. Perhaps even more so in space, where infrastructure is so costly and so vital.

Anonymous said...

Many times, the object of a war was to capture something valuible; land, resources, people (labor), or treasure;in space warfare, those objectives would also seem to be paramount. Say, for example, the colonies on Callistro get cut off from Earth,(maybe a new Great Depression causes a drastic delay in space launches), and some of those colonies decide that taking control of all the other colony sites would insure their survival; ground warfare on an airless Antarctica...

Ferrell

Tony said...

I think one of the fundamental features of war is that the destruction is aimed at people, their organizations, and their instrumentalities, with the purpose of enabling or preventing the taking of something valuable (land, people, goods, trade routes). The valuable things themselves you don't want to destroy. Fighting in space or on other planets isn't going to change that.

Thucydides said...

Warfare in the age of the Condottieri might make a good model. You don't want to actually destroy the objectctive, and the fighters (having control of lots of valuable resources in the form of ships, vehicles, equipment and trained manpower) might be a bit reluctent to expend it without a very good reason.

If the fighting troops are mercenaries, then the model becomes more exact (and today's enemies might become tomorrow's partners, so you don't want to cause too much mayhem amongst the enemy troops).

The modalities might change (in space warfare, Cyber capabilities and engineering troops both ground and naval might be much more valuable than Marines and Laserstars), but the overarching reasons for resorting to war will remain the same so long as we are human.

Tony said...

Well, Machiavelli's take on condotierri warfare was slanted to say the least. Mercenary rank and file made up only parts of the armies, and the military contractors themselves tended to form pretty strong joint venture relationships with their customers.

What I could see happening is that suppliers of physical means might also supply the technical personnel to handle maintenance and repair.

Thucydides said...

What I could see happening is that suppliers of physical means might also supply the technical personnel to handle maintenance and repair.

True enough. We see this now in various third world military forces where the nation can afford (or decides)to spend astronomical amounts of money on up to date military hardware that no one can understand or maintain. Tank parks and air bases become the homes of foreign contractors keeping the equipment at least serviceable, but not involved in the actual operations of the military.

From there it is only a short jump to contracting operators to use the equipment on your behalf. Expensive equipment like ships was contracted out during the Italian renaissance, something that became a sore point during the Battle of Lepanto; when the south wing was pulled out of position, Giovanni Andrea Doria was suspected of either having been bribed by the Turks, or wishing to preserve his huge investment in ships by avoiding pitched battle. (How true any of this was may never been known, it is more likely he was simply outmanouevred by his opponent).

This would make an interesting subtext to any military story using these conventions, are the contractors/mercenaries going to stay loyal or has the other side somehow subverted them?

Tony said...

Problem with the "mercs go to the highest bidder trope" is that it only works for mercs making a big play -- either for state control themselves or funding retirement. Up until that point, there's considerable value in loyalty, because nobody likes a fink.

Anonymous said...

If a merc gets a reputation for switching sides if the price is right, people stop hiring him and he starves to death or stops being a merc... For a merc, breach of contract can get him shot.

Ferrell

Thucydides said...

The unreliable mercenary/ switching sides for the right price trope is there because it happened enough times since ancient times to become ingrained.

Mercenaries who acted this way were indeed playing with fire, so it probably happened less often than we think, but even showing up a bit late, not following orders exactly or just being driven off by a superior force was often enough for the suspicious eye to be cast in your direction as a mercenary commander.

Tony said...

Thucydides:

The unreliable mercenary/ switching sides for the right price trope is there because it happened enough times since ancient times to become ingrained.

Funny thing is, the unreliable political ally is much more common in reliably narrated history.

Isaac said...

I have a couple of suggestions that I've put up in a different section. But no-one reads them there, which makes writing them pointless, so I've copied them over to here as it's more recent. Sorry if that's bad etiquette. My suggestions do work as cheap combat options as well though, I think. Just a mix of nukes, asteroids and kinetics, which I'd expect you'd find to hand early on in a conflict.

My first suggestion is for a killer bus (or just a swarm of missiles, whichever's more efficient)to be fired by coil gun from a ship already going incredibly fast, and to be guided in towards the enemy. After an initial explosion bursts open its payload (hundreds of guided soda cans, possibly with additional rubble), spread across the general trajectory of their ship, a nuke, left near behind, would go off.

While presumably ships would be well protected against radiation, the blast may damage some of their sensors. What it would certainly do is up the background radiation behind the incoming payload, so as to make it essentially invisible.

The payload would then home in, immune to point defense; being small, erratic, incredibly fast and coming out of what is in essence a sun.

This, instead of camouflaging the missiles, camouflages the background to match them. Multiple killer buses/swarms, some with nukes, some without, compound the problems for defense - as you would have to destroy not only the stuff that will hit you, but also the stuff that will get near, as any of it could be a nuke about to blind you in a crucial moment.

Also, maybe you could use a nuclear blast to hide your own ship behind? Pockets of radiation that mask your signature (or your shutter) could be really useful, and would liven up empty space.

My other suggestion is to hide your ship inside an asteroid for a sneak attack. You would have to do all of your thrust and correction outside their sensor range, which would require a lot of accuracy. Also you'd have to come in on a believable trajectory, ideally from the opposite direction to what they expect. A sufficiently large asteroid could easily hide the heat signature of an AI or skeleton crew ship. At the closest point you'd suddenly pivot and hit them where it hurts with a laser. Maybe one of those x-ray single shot ones? Ouch. This trick would only work once though, they'd learn to blow up all near flying asteroids just in case.

A different asteroid scenario is one where you catapault (with a planet) a whole bunch of rocks at the enemy, and hide ships in only a few of them. They'd know they were being attacked, but wouldn't know which rocks to destroy. If they targeted an asteroid with a ship inside it, it would return fire and evade, turning into a normal attack. They wouldn't know how many ships were attacking however until they'd destroyed every rock. Their tactic may then be to give each only a little fire to see if it responds; I can imagine a tense scene with a commander accepting enemy fire in an attempt to fool them into believing his ship is only an asteroid. Then of course you'd turn and hit them with everything. More boringly, but with less casualties, this kind of attack could be done with droids.

Anyway, those are some of my thoughts, hope someone reads them.

Anonymous said...

Some intersting stuff there.

I like the nuke idea - if your missiles were designed right you could even use the nuke shockwave for a little thrust ala Orion.

The dummy kinetic also sounds like it would work. If you had 1000 projectiles 90% were just "rocks" and 10% "missiles" How would the Laserstar know which to shoot down? It might reduce the cost of the saturation point quite a bit.

(SA Phil)

Tony said...

SA Phil:

"The dummy kinetic also sounds like it would work. If you had 1000 projectiles 90% were just "rocks" and 10% 'missiles' How would the Laserstar know which to shoot down? It might reduce the cost of the saturation point quite a bit."

To be effective, your decoys would have to have the same mass, density, shape, and albedo as the real thing. Otherwise you could process through the swarm quickly with relatively low energy pulses and see how they react. The minority set, however it reacts differently, becomes your target class.

BTW, when suggest balloon decoys as effective penetration aids against ballistic missile defenses, just point out to them that the mid-course "vacuum" still has enough atmosphere molecules in it to effect balloons noticeably more than reentry vehicles. It turns out that the only effective decoy is one that has all of the gross physical characteristics of a real, armed RV. At which point you might send over real, armed RVs instead of physically identical decoys.

The same logic applies with swarming kinetic attack against a defending laserstar. Might as well put navigation sensors and propulsion in every item in the swarm, unless the marginal cost of these systems is just too high.

Anonymous said...

Tony,

To be effective, your decoys would have to have the same mass, density, shape, and albedo as the real thing. Otherwise you could process through the swarm quickly with relatively low energy pulses and see how they react.
-----------

Anything that slows down the traget's laser shooting defense is helpful - its probably not necessary to absolutely defeat it with a dummy.

Regardless it might be easier to make a dummy with similar mass and outer casing without a propulsion+energy source, by filling it with some material that has similar density.

The saved costs might be in the manufacturing rather than materials end. These high tolerance propulsive technologies cost a lot of money.

The electronic sensor suite might be a lot cheaper, considering the trends we have seen in the last 50 years.

(SA Phil)

Anonymous said...

=Milo=



Isaac:

"What it would certainly do is up the background radiation behind the incoming payload, so as to make it essentially invisible."

"Also, maybe you could use a nuclear blast to hide your own ship behind? Pockets of radiation that mask your signature (or your shutter) could be really useful, and would liven up empty space."

The radiation only exists for a brief moment. There's nothing in vacuum for fallout to stick to, so it gets blasted outward at incredible speeds.

If your sensors survive being fried by the blast, they'll be back to normal operation too fast for your missiles to close the distance.

Anonymous said...

Milo

The radiation only exists for a brief moment. There's nothing in vacuum for fallout to stick to, so it gets blasted outward at incredible speeds.
----

What if you were to use something that lasts a bit longer and spreads out- Such as a plasma?

IE turn the missile around and "jetwash" the target.

(SA Phil)

Tony said...

SA Phil:

"Anything that slows down the traget's laser shooting defense is helpful - its probably not necessary to absolutely defeat it with a dummy.

Regardless it might be easier to make a dummy with similar mass and outer casing without a propulsion+energy source, by filling it with some material that has similar density.

The saved costs might be in the manufacturing rather than materials end. These high tolerance propulsive technologies cost a lot of money.

The electronic sensor suite might be a lot cheaper, considering the trends we have seen in the last 50 years."


All true. I'm just pointing out that there is such a thing as a naive -- and consequently ineffective -- application of decoy technology.

Anonymous said...

I also had a though about a missile that absorbed less of a laser's energy.

If some of the energy passed through - presumably it would take longer to kill/move (force to course deviate beyond correctable limits)

Perhaps a high optical quality Diamond/Quartz missile filled with a fluid that has high optical tranferance (for use as propellant)

With the thought that slowing down the destruction of the cloud of impending kinetic death means the size of the cloud that will work becomes smaller.


(SA Phil)

Rick said...

Isaac - your comments are entirely welcome and appropriate!

The same logic applies with swarming kinetic attack against a defending laserstar. Might as well put navigation sensors and propulsion in every item in the swarm, unless the marginal cost of these systems is just too high.

Agree. I originally favored kinetic fragmentation weapons, but I was still thinking in a midcentury mind frame where metal is cheap, control systems expensive. (Put another way, I pictured a target seekers as roughly the size/mass of a torpedo.)

But as space payloads, 'soda cans of death' are not all that much more expensive than a similar mass of rocks, and a far higher proportion are likely to hit.

That said, if the target seekers are delivered by bus, fragments of the bus can add clutter, and flare/chaff type decoys can make the separation process harder to track, adding modestly but non-trivially to the challenge of identifying and engaging the target seekers.

Anonymous said...

Rick said:"That said, if the target seekers are delivered by bus, fragments of the bus can add clutter, and flare/chaff type decoys can make the separation process harder to track, adding modestly but non-trivially to the challenge of identifying and engaging the target seekers."

Why stop there? Make the bus a jammer platform that interferes with the target's sensors. And on a colision course...

Ferrell

Anonymous said...

If the advantage of the bus is basically propellant/propulsive in nature,

The soda can of death wouldnt need a bus- would it?

Unless the bus were the low thrust/high ISP stage - while the missile carried only a simple high thrust propulsion system.

(SA Phil)

Tony said...

SA Phil:

"The soda can of death wouldnt need a bus- would it?"

You would use a bus to launch multiple soda cans on one booster, the same way 3-10 MIRVs could be launched on a single ICBM. Except is would be hundreds of soda cans instead of several nuclear devices. Once the booster burns out and separates, the bus waits until an optimum point in the trajectory, then spins up and distributes soda cans centrifugally.

Rick said...

Pretty much what I had in mind

The point about a high-ISP stage is also relevant. The Soda Can of Death has only a chemfuel deflect motor, probably with no more than 1 km/s of delta v, if that much. So whatever deploys it has to provide most of the closing velocity.

Unless the engagement geometry does the heavy lifting, as it might, in which case you simply release it like a guided bomb.

Rick said...

Addendum that yes, the bus might also carry penetration aids, such as a relatively low-power laser to dazzle sensors.

Isaac said...

As an improvement on my blinding nuke idea, perhaps you could use a whole series of blasts? Or time the blast/s to cover a change of direction for the swarm?

Or, would it be possible to have missiles with lasers? They would fire a continuous beam directly at the target, so that if a laserstar locked on to them, it would be blinded. What happens when two lasers fire at each other?

Rick said...

Belated response:

What happens when two lasers fire at each other?

That is a Really Interesting Question.

My guess is that they mutually degrade until the less powerful one is knocked out entirely. Probably the ultimate Lanchesterian engagement.

Though it may also be the case that, above some minimal threshold, they knock each other out of action.

Anonymous said...

Or an internal 'optic circut-braker' is tripped and the laser shuts down and incoming light is deverted away from the internal mechinisms...just like in the real-life ABL.

Ferrell

Saint Michael said...

Today, when a US Space Force is a real (if implausible!) military branch, Russia is getting bitterly defensive about the loss of its Glory Days, and China now seems determined to start a new Space Race, it's a whole new Solar System.

I would have been highly skeptical of a militarized space future even a few years ago, but "it's hard to make predictions, especially about the future". So here's my updated vision.

First, no dedicated Space Warships in the PMF. Spacecraft are too expensive to waste one on a large vessel that sits around waiting for something big to happen. Ditto Laserstars, should they come to pass.

What you describe as patrol cutters or cruisers will dominate for power projection, as they're probably all that's all that is needed for gunboat diplomacy IN SPACE. Orbital habitats and industrial facilities are very fragile, and any purpose-built military vessel will likely outmatch anything civilian. Of course, Kzinti Lesson and all that, you're never truly unarmed in space.

If a real shooting war breaks out, expect the cutters to be repurposed as control ships for drones and smart missiles, possibly they'll be built so that add-on comm and computer modules can simply snap into place. Drones can be armed or sensory, or both. By default it's likely that any drone, or even most missiles, will do some intel-gathering to laser back to the Any large scale transport of kinetics and drones can be covered by standard cargo ships. Propellant tankers will probably figure heavily into any constellation's logistics.

As far as laser weapons, they might not be humongous death rays with ranges in the light-seconds, but there will be dedicated laser drones, probably transportable by cargo freighter. What specific laser technologies they use is not foreseeable. But they'll still need extensive heat management and rejection to do their jobs. Also, mirror drones (with their own active cooling systems) will need to be delivered in the same load.

There's not much chance that a colony or reasonable collection of colonies, will have enough vessels to outmass those available to Earth in the PMF. But a proxy war between Earth powers is another story, and not implausible. So you might see armed merchantmen throwing dumb rocks and sniping at each other.

Of course when you have orbital speeds and high-energy industrial tools available, an easy one-hit kill on a habitat makes all warfare Mutual Assured Destruction. So whatever warfare occurs will either be limited, or very very bloody.

«Oldest ‹Older   1 – 200 of 201   Newer› Newest»